Die Gerüchte über den ersten direkten Nachweis von Gravitationswellen werden immer intensiver. Eigentlich bringt es nicht viel, über Gerüchte zu berichten. Aber in diesem Fall sind sie mittlerweile sehr konkret und haben es auch schon auf die Homepage von “Science” geschafft. Es lohnt sich daher, ein wenig genauer hinzuschauen. Vor allem aber kann man zumindest ein wenig darüber lernen, worum es bei all der Aufregung eigentlich geht.

(Nachtrag: Am 11. Februar 2016 wurde offiziell bestätigt: Der direkte Nachweis von Gravitationswellen ist gelungen. Und warum das wirklich eine enorm große Sache ist, habe ich hier erklärt)

Überall Wellen im Raum - oder doch nicht? (Künstlerische Darstellung: Henze, NASA)

Überall Wellen im Raum – oder doch nicht? (Künstlerische Darstellung: Henze, NASA)

Was sind Gravitationswellen?

Gravitationswellen sind eine Vorhersage aus Albert Einsteins Allgemeiner Relativitätstheorie und eines der wenigen Phänomene, das noch nicht direkt nachgewiesen worden ist. Laut Einstein krümmen Massen die Raumzeit und diese Krümmung beeinflusst die Bewegung von Himmelskörper. Wir nehmen diese Beeinflussung als Gravitationskraft wahr. So weit, so klar und diese Aussage wurde schon vor fast 100 Jahren und seitdem immer wieder im Experiment nachgewiesen.

Ein wichtiger Punkt bei Einsteins Theorie ist die Geschwindigkeit, mit der sich Veränderungen der Krümmung ausbreiten. Oder anders gesagt: Die Geschwindigkeit der Gravitation. In der alten Theorie von Newton wurde dazu ja nichts festgelegt; laut Newton war Gravitation unendlich schnell – was aber weder begründet noch logisch sinnvoll war. Einstein konnte zeigen, dass sich auch die Gravitation nur so schnell wie das Licht ausbreiten kann. Würde also zum Beispiel plötzlich die Sonne verschwinden, würde sich die Veränderung in der Raumzeitkrümmung von dort aus in alle Richtungen ausbreiten und die Erde 8 Minuten später erreichen – so lange dauert es, um die 150 Millionen Kilometer Abstand mit Lichtgeschwindigkeit zurück zu legen.

Solche Gravitationswellen werden aber nicht nur erzeugt, wenn ein Stern verschwindet. Immer wenn sich Massen beschleunigt bewegen, erzeugen sie Gravitationswellen. Sie sind um so stärker, je massereicher und dichter die beteiligten Objekte sind. Wenn zum Beispiel zwei Neutronensterne oder zwei schwarze Löcher einander umkreisen, dann erzeugen vergleichsweise starke Gravitationswellen.

Das sagt zumindest die Theorie. Direkt beobachtet hat man solche Gravitationswellen bis jetzt noch nicht. Indirekt dafür aber schon: Im Jahr 1974 haben die beiden Astronomen Russell Hulse und Joseph Taylor zwei einander umkreisende Neutronensterne entdeckt und konnten beobachten, dass sie einander immer näher kommen. Sie müssen also irgendwie Bewegungsenergie verlieren – und die Menge und Rate mit der sie das tun entspricht genau dem, was man erwartet, wenn sie diese Energie durch die Abstrahlung von Gravitationswellen verlieren würden. Für diesen ersten indirekten Nachweis haben die beiden immerhin 1993 den Nobelpreis für Physik bekommen. Aber noch besser wäre es natürlich, wenn man die Gravitationswellen auch direkt beobachten könnte.

Wie beobachtet man Gravitationswellen?

Das Problem beim direkten Nachweis von Gravitationswellen ist die Stärke des Effekts. Die Raumzeit stellt man sich ja gerne als “Gummituch” vor; in Wahrheit ist sie aber extrem schwer verformbar. Das Maß, das in der Physik angibt wie gut sich ein Material verformen lässt, heißt “Elastizitätsmodul”. Bei Gummi sind das zum Beispiel 0,1 Gigapascal. Holz ist 100 Mal schwerer zu verformen und hat einen Elastizitätsmodul von 10 Gigapascal. Stahl ist 2000 Mal stärker als Gummi – die Raumzeit aber ist viel, viel steifer. Hier beträgt der Elastizitätsmodul 1024 Gigapascal; sie ist also 10 Quadrillionen mal schwerer zu verformen als Gummi.

Jetzt wird auch klar, wieso man bei diesem Thema immer wieder auf schwarze Löcher oder Neutronensterne trifft. Nur solche extrem dichten und massereichen Objekte können die Raumzeit so sehr verformen, dass wir überhaupt eine Chance haben, davon etwas mitzubekommen. Um Gravitationswellen zu messen, braucht man einen Detektor. Kommt dann von außen aus dem Weltraum eine solche Welle auf uns zu und läuft durch die Erde und damit den Detektor hindurch, dann wird der gesamte Raum rhythmisch komprimiert und gestreckt. Vereinfacht gesagt: Der Detektor wird abwechselnd ein wenig länger und kürzer. Mit ihm aber auch der gesamte Rest der Raumzeit, weswegen es ein wenig knifflig ist, diesen Effekt nachzuweisen.

Nördliche LIGO-Röhre - recht unscheinbar für ein so dramatisches Experiment (Bild: Umptanum, CC-BY-SA 3.0)

Nördliche LIGO-Röhre – recht unscheinbar für ein so dramatisches Experiment (Bild: Umptanum, CC-BY-SA 3.0)

Momentan benutzt man bei den Experimenten sogenannte Interferometrische Detektoren. Das empfindlichste Gerät ist derzeit LIGO (Laser-Interferometer-Gravitationsobservatorium) in den USA. Die Anlage besteht aus zwei vier Kilometer langen Röhren, die im rechten Winkel zueinander angeordnet und am Ende miteinander verbunden sind. Im Kreuzungspunkt steht ein Laser, der einen Strahl gleichzeitig in beide Röhren schickt. An deren Enden sind Spiegel montiert, die den Laserstrahl zurückwerfen. Das Licht wird mehrmals hin und her reflektiert, so dass es insgesamt eine deutlich größere Strecke als nur die vier Kilometer zurück legt. Die Anlage ist so gebaut, dass die Strecken in den beiden Armen exakt gleich lang ist. Am Ende sollte also das Licht auch aus beiden Armen wieder exakt gleichzeitig am Ausgangspunkt eintreffen. Modifiziert man die Position der Spiegel, kann man das System aber auch so einstellen, dass ein Strahl ein winzig bisschen früher/später ankommt als der andere. Treffen beide im Detektor zusammen, trifft ein Wellenberg der einen Lichtwelle auf ein Wellental der anderen und beide löschen sich aus. Dank dieser Interferenz misst der Detektor im Idealfall also gar kein Licht mehr.

Wenn jetzt nun aber eine Gravitationswelle die Anlage durchläuft, dann wird sie komprimiert bzw. gestreckt. Die Strecke, die das Licht durchlaufen muss, ändert sich. Und da die beiden Röhren im rechten Winkel zueinander stehen, werden sie von der Gravitationswelle unterschiedlich stark gestaucht bzw. gestreckt und das Licht legt unterschiedliche Strecken zurück. Die Feinabstimmung passt nun also nicht mehr; die Laserstrahlen löschen sich bei der Ankunft nicht mehr aus. Es gibt ein Signal und Nobelpreise für alle.

So einfach ist die Sache aber leider nicht. Wie gesagt: Der Effekt ist sehr gering und man muss mit extremer Genauigkeit arbeiten. Was man auch tut; bei LIGO schafft man es, Längenunterschiede in den beiden Armen zu messen, die 1000 Mal kleiner sind als der Durchmesser eines Protons! Aber das reicht gerade mal, um die stärksten Gravitationswellen zu detektieren und birgt natürlich auch jede Menge Möglichkeiten, andere Dinge zu messen. Zur Sicherheit hat LIGO daher auch noch eine zweite Anlage in 3000 Kilometer Entfernung eingerichtet. Eine Gravitationswelle, die sich ja nur mit Lichtgeschwindigkeit bewegt, kommt dort ein klein wenig später an. Messen beide Observatorien die gleiche Längenänderung und passt auch noch der Abstand zwischen den Signalen, dann kann man sich ziemlich sicher sein, tatsächlich etwas beobachtet zu haben.

Hat man etwas beobachtet?

Und hat man nun etwas beobachtet? Seit Wochen machen entsprechende Gerüchte im Internet die Runde. Es ist auf jeden Fall nicht unwahrscheinlich. LIGO sollte zumindest prinzipiell in der Lage sein, Gravitationswellen zu beobachten. Wenn die Vorhersagen der Relativitätstheorie stimmen, dann müsste der Detektor ausreichend sensibel sein, um zumindest die stärksten Ereignisse zu messen, die zum Beispiel bei der Kollision zweier schwarzer Löcher entstehen. Außerdem hat man erst letztes Jahr im Herbst die Genauigkeit des Observatoriums noch einmal erhöht. Es spricht also nichts dagegen, dass man dort erfolgreich ist und die Wissenschaftler rechnen natürlich auch damit.

Ebenfalls letztes Jahr im Herbst hat der Physiker Lawrence Krauss in einem Tweet behauptet, es gäbe Hinweise auf eine erste Beobachtung. Im Januar 2016 twitterte er erneut und gab bekannt, dass er eine Bestätigung der Hinweise erhalten hatte. Aber Gerüchte sind eben Gerüchte und von offizieller Seite gab es keine Aussage.

Vor ein paar Tagen wurde dann schließlich die Email von Clifford Burgess, eines anderen Physikers veröffentlicht. Darin erklärt er ein wenig detaillierter, was man bei LIGO beobachtet haben will und weist darauf hin, dass die Entdeckung am 11. Februar 2016 offiziell bekannt gegeben werden soll.

In der Email von Burgess steht:

Hi all, the LIGO rumour seems real, and will apparently come out in Nature Feb 11 (no doubt with press release), so keep your eyes out for it. Spies who have seen the paper say they have seen gravitational waves from a binary black hole merger. they claim that the two detectors detected it consistent with it moving at speed c given the distance between them, and quote an equivalent 5.1 sigma detection. the bh masses were 36 and 29 solar masses initially and 62 at the end. Apparently the signal is spectacular and they even see the ring-down to kerr at the end.

Woohoo! (I hope)

Wie gesagt. Es gibt immer noch keine offizielle Bestätigung, aber am 11. Februar wissen wir hoffentlich mehr. Bis dahin ist es aber ganz interessant sich anzusehen, was da im Detail behauptet wird und wie eine echte Detektion von Gravitationswellen aussehen könnte.

Was kann man beobachten?

In der Email geht es um einen sogenannten “binary black hole merger”, also eine Verschmelzung von zwei schwarzen Löchern. Dass zwei schwarze Löcher einander umkreisen wäre eigentlich nichts ungewöhnliches. Wir wissen, dass die Mehrheit der Sterne Teil von Doppel- oder Mehrfachsternsystemen ist. Ob ein Stern am Ende seines Lebens zu einem roten Riesen, einem Neutronenstern oder einem schwarzen Loch wird, hängt nur von dessen Masse ab. Unsere Sonne wird ein roter Riese werden; schwerere Sterne können schwarze Löcher werden. Wenn nun zwei schwere Sterne Teil eines Doppelsternsystems sind, können sie irgendwann als einander umkreisende schwarze Löcher enden. Wir haben schon einige Mehrfachsysteme entdeckt, in denen einander Sterne, schwarze Löcher und Neutronensterne umkreisen; eines mit zwei schwarzen Löchern war allerdings noch nicht dabei. Aber geben müsste es sie eigentlich und wenn es sie gibt, dann sind sie ideale Kandidaten für den Nachweis von Gravitationswellen.

So wie beim Doppelpulsar von Hulse und Taylor würden auch die schwarzen Löcher Gravitationswellen abstrahlen und dabei Energie verlieren. Sie würden einander immer näher kommen, miteinander kollidieren und schließlich zu einem einzigen schwarzen Loch verschmelzen. Wie das genau abläuft, kann man zumindest theoretisch berechnen und/oder am Computer simulieren und auch die Art der Gravitationswellen die dabei ausgestrahlt werden, lässt sich vorhersagen.

Entsprechende Facharbeiten gibt es genug, zum Beispiel eine, die Anfang Januar 2016 publiziert worden ist (“Music from the heavens – Gravitational waves from supermassive black hole mergers in the EAGLE simulations”). Darin findet man auch eine schöne Grafik, die demonstriert, was laut Gerücht angeblich bei LIGO beobachtet worden sein soll:

Vereinfacht gesagt erkennt man hier die Stärke und Rate, mit der die Raumzeit schwingt. Zuerst umkreisen die schwarzen Löcher einander noch außerhalb des “Last Stable Circular Orbit (LSCO)”, also der letzten stabilen Bahn. Je näher sie aber einander kommen, desto stärker werden die Gravitationswellen und desto schneller folgen sie aufeinander; das ist die “Merger”-Phase, in der sie verschmelzen. Danach entsteht ein neues schwarzes Loch, das sich aber erst noch “einrichten” muss. Kurz nach der Verschmelzung hat es noch nicht die optimale Form. Es befindet sich noch nicht im Gleichgewicht und während es dieses Gleichgewicht findet, gibt es weitere Gravitationswellen ab. Das ist die Phase, die im Diagramm als “Ringdown” bezeichnet wird. Je näher das schwarze Loch an der energetisch günstigen Form ist, desto schwächer werden die Wellen, bis sie schließlich ganz verschwinden.

So weit die Theorie. Laut der Email von Burgess will man bei LIGO genau so ein Ereignis beobachtet haben. Die beiden schwarzen Löcher, die miteinander verschmolzen sein sollen, hätten die 36fache bis 29fache Masse der Sonne. Am Ende wäre daraus ein schwarzes Loch mit der 62fachen Sonnenmasse entstanden. Die LIGO-Daten sollen genau den Vorhersagen für so ein Ereignis entsprechen und man hätte sogar die “Ringdown”-Phase am Ende der Verschmelzung detektiert. Außerdem hätten beide LIGO-Observatorien die Verschmelzung registriert und zwar genau in dem zeitlichen Abstand der erwartet wird, wenn die Gravitationswellen sich mit Lichtgeschwindigkeit von einem zu anderen bewegen.

Interessant auch der Unterschied in den Massen. 36 plus 29 ergibt ja 65 und nicht 62. Das bedeutet, dass bei der Verschmelzung der beiden schwarzen Löcher ganze drei Sonnenmassen verschwunden sein müssen. Aber die sind natürlich nicht verschwunden: Das ist genau die Energie, die in den Gravitationswellen steckt. Das macht vielleicht noch einmal besonders deutlich, was für ein enorm hochenergetisches Ereignis so eine Verschmelzung schwarzer Löcher ist…

Zusammengefasst: Die angebliche Beobachtung von LIGO entspricht genau dem, was LIGO zu beobachten in der Lage sein sollte und was man sich beim LIGO-Experiment erwartet hat. Das bedeutet natürlich noch nicht, dass es auch wirklich beobachtet wurde. Vielleicht handelt es sich wirklich nur um ein Gerücht, das außer Kontrolle geraten ist. Vielleicht handelt es sich um einen internen Test der Detektoren mit einem künstlichen Signal der von Außenstehenden falsch verstanden wurde. Vielleicht um etwas ganz anderes.

Künstlerische Darstellung der Verschmelzung zweier schwarzer Löcher (Bild: NASA)

Künstlerische Darstellung der Verschmelzung zweier schwarzer Löcher (Bild: NASA)

Wenn es sich tatsächlich um den ersten direkten Nachweis von Gravitationswellen handelt, werden wir es bald erfahren. Und wenn nicht? Dann wird es vermutlich trotzdem nicht mehr allzu lange dauern. Es wäre extrem seltsam, wenn gerade diese Vorhersage der allgemeinen Relativitätstheorie nicht stimmen würde. Es wäre seltsam, wenn nach dem indirekten Nachweis der direkte Nachweis nicht gelingen sollte. Es wäre seltsam, wenn die Gravitationswellen ausbleiben obwohl alles was wir bisher nachweislich über das Universum wissen einstimmig vorhersagt, dass es sie geben muss. Und auch wenn das Universum manchmal seltsam ist: In diesem Fall traue ich mich zu behaupten, dass es sich früher oder später an die Vorhersagen halten wird.

Der direkte Nachweis von Gravitationswellen wird früher oder später (wahrscheinlich früher) kommen. Wenn nicht in dieser Woche, dann in naher Zukunft. Und wenn es soweit ist, wisst ihr immerhin schon darüber Bescheid, wie LIGO funktioniert und was man sich unter einem “Merger” und einem “Ringdown” vorstellen kann 😉

Weitere Informationen

Ich habe früher schon über Gravitationswellen geschrieben und wer noch mehr wissen will, wird hier hoffentlich fündig:

Kommentare (247)

  1. #1 haarigertroll
    8. Februar 2016

    Ich finde es ja fast unglaublich, dass man auf der Erde etwas bauen kann, das so präzise messen kann. Würde mich interessieren, wie die thermische Dehnung, Mikroseismik und sonstige Störgrößen kompensiert werden!

  2. #2 Velatus
    Milchstrasse
    8. Februar 2016

    Würde es irgendwas Bedeutungsvolles bedeuten, wenn die Gravitationswellen nun nachgewiesen wären? Irgendeine Erkenntnis?

  3. #3 Bullet
    8. Februar 2016

    @FF: Lustig, wie so verschiedene Themen zusammenkommen: da war doch letztens nebenan dieses Pamphlet verlinkt worden, in dem – lol – 200 “Beweise” für eine flache Erde präsentiert wurden. Weil ichs noch in Kopp, äh, Kopf hatte, fiel mir auf: diese exakt geraden Röhren mit den Signalstrecken müßten doch dann an beiden Enden etwas höher über dem Erdboden platziert sein als in der Mitte (vorausgesetzt, der Boden unter den Röhren wäre einigermaßen eben), um die Erdkrümmung auszugleichen. Ich hab in deinem Artikel nichts dergleichen gesehen (weils natürlich nicht so wichtig ist), aber bist du bei der Recherche über LIGO über eine derartige Erwähnung gestolpert?

  4. #4 Martin
    8. Februar 2016

    haarigertroll, daher gibt es zwei Systeme mit einem gewissen Abstand. Seismische Ereignisse sind langsamer als Licht und thermische Effekte dürften nur jeweils in einem der beiden Instrumente gemessen werden. Erst wenn man in beiden Systemen ein Ereignis messen kann und die zeitliche Differenz der Zeit entspricht, die Licht für die Strecke zwischen den Systemen benötigt, kann man evtl. von Gravitationswellen ausgehen.

  5. #5 MartinB
    8. Februar 2016

    Hast du für den E-Modul der Raumzeit eine seriöse Quelle? ich habe da bisher immer nur etwas windige Quellen gefunden und hätte gern mal ne genaue Erklärung, wie diese Zahl zu Stande kommt.

  6. #6 Alderamin
    8. Februar 2016

    @Florian

    bei LIGO schafft man es, Längenunterschiede in den beiden Armen zu messen, die 1000 Mal kleiner sind als der Durchmesser eines Protons!

    Interessant finde ich auch, wie man mit Laserlicht Abstände messen kann mit einer Auflösung weit unterhalb der Lichtwellenlänge. Kann einer der Physiker hier das nachvollziehen und erläutern? Martin?

  7. #7 Bullet
    8. Februar 2016

    @Velatus: mais naturellement.
    a) schon wieder eine Einstein-Vorhersage, die experimentell überprüft werden kann.
    b) – daraus folgend – ein Bausteinchen, das sich nahtlos in unser Weltbild einfügt und dadurch ein Indiz dafür ist, daß wir uns bisher nicht gar so großen Schwachsinn ausgedacht haben
    c) die Lichtgeschwindigkeit ist wirklich eine fundamentale Konstante
    d) du kannst erst dann ein funktionierendes Antigrav-Gerät bauen, wenn du weißt, was du eigentlich manipulieren willst
    e) Grundlagenforschung ist immer wichtig
    f) Radiowellen wurden von Maxwell vorhergesagt. Hertz hat sie gefunden. Jeder, der damals “Würde es irgendwas Bedeutungsvolles bedeuten, wenn die Funkwellen nun nachgewiesen wären? Irgendeine Erkenntnis?” als Leserbrief an irgendeine Zeitung geschrieben hätte, dürfte sich völlig gerechtfertigt “Idiot” nennen.

    Reicht das für den Anfang?

  8. #8 Bullet
    8. Februar 2016

    @Alderamin:

    Interessant finde ich auch, wie man mit Laserlicht Abstände messen kann mit einer Auflösung weit unterhalb der Lichtwellenlänge. Kann einer der Physiker hier das nachvollziehen und erläutern? Martin?

    Hö? Interferometrie? Kennst du doch.

  9. #9 roel
    *****
    8. Februar 2016
  10. #10 Florian Freistetter
    8. Februar 2016

    @Velatus: Dazu kann ich dir diesen Podcast empfehlen (der auch im Text verlinkt ist): https://scienceblogs.de/astrodicticum-simplex/2014/11/07/sternengeschichten-folge-102-gravitationswellenastronomie/

  11. #11 Florian Freistetter
    8. Februar 2016

    @MartinB: “Hast du für den E-Modul der Raumzeit eine seriöse Quelle?”

    Das hat ein Wissenschaftler bei dem Workshop in Schweden erzählt, bei dem ich mal war: Patrick Sutton; hier: https://scienceblogs.de/astrodicticum-simplex/2013/05/28/wo-sind-die-gravitationswellen/?all=1 (Wie er das berechnet hat, hab ich damals aber nicht mitgeschrieben und erinnere mich leider auch nicht mehr)

  12. #12 Fellianer
    Europa, Erde, Sonennsystem
    8. Februar 2016

    @Bullet:

    “To ensure a perfectly flat, straight, and level beam path, the Earth’s curvature (more than a vertical meter over each 4 km arm) was countered by GPS-assisted earth-moving and high-precision concrete work. Reinforced concrete floors 75 cm thick under the interferometers also minimize leak-through of seismic vibrations. […]”

    Von der LIGO Homepage.

  13. #13 Alderamin
    8. Februar 2016

    @Bullet

    Klar, mir ist nur nicht klar, warum sich damit das Auflösungsvermögen für eine gegebene Lichtwellenlänge austricksen lässt. Man kann ja generell keine kleineren Objekte abbliden, als die Lichtwellenlänge, und mit Optiken nicht schärfer auflösen, als das Dawes-Limit, deswegen wundert mich ein wenig, dass man mit Interferometrie beliebig (?) scharf messen können soll.

  14. #14 MartinB
    8. Februar 2016

    @Florian
    Danke, ich schau mal, ob ich mit dem Namen mehr herausbekomme.

    @Alderamin
    Die Auflösung mit der Lichtwellenlänge bezieht sich auf die Auflösung senkrecht zur Ausbreitung (so wie bei ner Linse oder so). Hier haben wir ja eine Messung in Ausbreitungsrichtung, da gilt die Abbe-Gleichung nicht. Es werden ja nur Phasenunterschiede detektiert.

  15. #15 Alderamin
    8. Februar 2016

    @MartinB

    Danke!

  16. #16 Krypto
    8. Februar 2016

    @Florian:
    Danke für die ausführliche Info und Einordnung!

    Eine Gravitationswelle, die sich ja nur mit Lichtgeschwindigkeit bewegt, kommt dort ein klein wenig später an.

    Würde ich ändern:
    “höchstwahrscheinlich ein klein wenig früher oder später an.”
    Höchstwahrscheinlich deshalb, weil es auch Konstellationen geben kann, bei denen die Wellen exakt gleichzeitig durch beide Detektoren laufen können.

  17. #17 Velatus
    Milchstrasse
    8. Februar 2016

    @Bullet DANKE! Ist immer schwierig das in so komprimierter Form zu finden. Klasse!

  18. #18 Artur57
    8. Februar 2016

    @Martin, Alderamin

    Man geht ja davon aus, dass die Gravitationswelle den einen Schenkel des L ein wenig streckt und den anderen ein wenig staucht. Das Licht hat dann einen kürzeren, bzw. längeren Weg. Wenn sich die Unterschiede auf eine halbe Wellenlänge summieren, wird dabei der Bereich von maximaler zu destruktiver Interferenz durchlaufen. Oder eigentlich nur eine viertel Wellenlänge, weil beide Schenkel zweifach durchlaufen werden. Oder nochmals reduziert, wenn die Schenkel mehrfach durchlaufen werden.

    Um nun noch feiner zu werden, müsste man Zwischenzustände erkennen können. Denke schon, das die das können.

    Ja nun, das klappt alles aber nur, wenn die Gravitationswelle nur die Baulichkeit verkürzt oder verlängert, nicht aber gleichzeitig die Wellenlänge des Lichts. Denn dann mäße man exakt nichts, auch wenn eine Gravitationswelle durchläuft. Wäre schon mal einen Gedanken wert, ob das so ist.

    Denn dass die Raumzeit die Wellenlänge eines Photons verändert, das akzeptieren wir ja mühelos, wenn wirt davon ausgehen, dass die Raumdehnung eine Strahlung von 2000 K auf 3 K abgekühlt hat, wie bei der Hintergrundstrahlung geschehen.

  19. #19 Till
    8. Februar 2016

    @florian Danke, das war mal wieder ein klasse Artikel! Insgesamt finde ich, dass seit Du Deine Blogaktivität im Januar reduziert hast zwar weniger aber dafür (für mich) interessantere Artikel erscheinen. Vielen Dank und weiter so!

  20. #20 Bullet
    8. Februar 2016

    @Velatus:
    echt? Na gut, vielleicht ist das Biotop hier ein etwas spezielles. Die Frage, die du gestellt hast, wird gerne von Leuten verwendet, die generell etwas gegen Forschung haben und bei jedem Stück Forschungsergebnis, bei dem nicht sofort ein neuer Fernsehsender oder ein noch geileres Smartphone hinten rausfällt, von “Verschwendung” lamentieren.
    Ich persönlich finde ja, aus der Liste ist Punkt b) da am wichtigsten. Denn es ist ja klar, daß das Universum schon ein, sagen wir, “funktionsfähiges” Gebilde ist, in dem alle Puzzleteile zueinander passen müssen. Wenn sich jetzt heftige Schwierigkeiten zeigen würden, die Gravitationswellen mit dem Rest der Physik zusammenzubringen, wäre es eine Überlegung wert, ob nicht irgendwo ein Fehler auch in den bereits als bekannt angenommenen Teilen der Weltbeschreibung vorliegt. Wie in einem großen Sudoku, beispielsweise. Wenn plötzlich eine Zahl nicht mehr dahinpaßt, wo sie eigentlich sein müßte, dann hast du vorher schon was falsch gemacht…

  21. #21 Bullet
    8. Februar 2016

    @Artur57:

    das klappt alles aber nur, wenn die Gravitationswelle nur die Baulichkeit verkürzt oder verlängert, nicht aber gleichzeitig die Wellenlänge des Lichts. Denn dann mäße man exakt nichts, auch wenn eine Gravitationswelle durchläuft. Wäre schon mal einen Gedanken wert, ob das so ist.

    Diese Gedanken gibts schon. Es gibt auch schon eine Antwort. Und jeder Effekt läßt sich andersrum verwerten.
    Hier ist es eher so, daß eine Gravitationswelle eine Lichtwelle erst staucht und dann wieder streckt. Im Ergebnis ist dann aber spätestens die Phasenlage eine andere. Und genau dazu ist dieser Versuchsaufbau bei LIGO gedacht.

  22. #22 Thomas Ahrendt
    8. Februar 2016

    An Bullett
    F) Radiowellen
    Vorhersagen sind schwierig, besonders wenn sie die Zukunft betreffen. Dann weißt du also schon, was man mit Gravitationswellen machen kann?? Erzähl doch mal.

  23. #23 Alderamin
    8. Februar 2016

    @Artur57

    Die Frage ist ziemlich gut. Die LIGO-Seite hat die Antwort: es kommt nicht auf die Wellenlänge oder die Zahl der Wellen in den Interferometerarmen an, sondern auf die Laufzeit, und die ist wegen der Konstanz der Lichtgeschwindigkeit nur von der Länge der Arme abhängig.

  24. #24 Alderamin
    8. Februar 2016

    @Thomas Arendt

    Z.B. kann man (nur) mit Gravitationswellen in die Zeit vor der Entstehung der kosmischen Hintergrundstrahlung 380000 Jahre nach dem Urknall schauen. Und halt die Verschmelzung von Schwarzen Löchern und Neutronensternen beobachten, was unsere Theorien über diese Objekte belegen oder widerlegen kann. Wir haben ja noch nie die Möglichkeit gehabt, Prozesse solcher Energien im Detail zu studieren.

  25. #25 Bullet
    8. Februar 2016

    @Thomas Ahrendt:

    Dann weißt du also schon, was man mit Gravitationswellen machen kann?? Erzähl doch mal.

    Du hast eine Stelle gefunden, an der ich behaupte, daß ich weiß, was man mit Gravitationswellen machen kann?? Erzähl doch mal.

  26. #26 Bullet
    8. Februar 2016

    Und natürlich: gugg doch mal e): “Grundlagenforschung ist immer wichtig”. Man weiß eben nie, was man mit solchen Erkenntnissen in der Zukunft anfangen kann. Aber ich würde mal wetten, daß es mir auch mit langem Nachdenken nicht gelingt, ein Grundlagenforschungsergebnis zu benennen, aus dem sich absolut nichts machen läßt.
    Von daher ist es mit Sicherheit ein Fehler, zu glauben, daß ein Ergebnis aus der Grundlagenforschung wohl zu nichts gut wäre (und erst recht, dieses Ergebnis deswegen zu verwerfen).

  27. #27 Herr Senf
    8. Februar 2016

    @MartinB #5
    Bin fündig geworden:
    Einsteinsche Feldgleichungen Guv = (8π G/c^4)*Tuv
    haben die Form einer Elastizitätsgleichung, in Analogie
    ist das Elastizitätsmodul G/c^4 ~ 10^-43 /N

    also ist der Raum extrem steif

  28. #28 Herr Senf
    8. Februar 2016

    @MartinB #5
    Bin fündig geworden:
    Einsteinsche Feldgleichungen Guv = (8π G/c^4)*Tuv
    haben die Form einer Elastizitätsgleichung, in Analogie
    ist das Elastizitätsmodul G/c^4 ~ 10^-43 /N

    also ist der Raum extrem steif

  29. #29 Karl Mistelberger
    8. Februar 2016

    > Alderamin, 8. Februar 2016
    >> bei LIGO schafft man es, Längenunterschiede in den beiden Armen zu messen, die 1000 Mal kleiner sind als der Durchmesser eines Protons!

    > Interessant finde ich auch, wie man mit Laserlicht Abstände messen kann mit einer Auflösung weit unterhalb der Lichtwellenlänge. Kann einer der Physiker hier das nachvollziehen und erläutern?

    Zum Verständnis der Genauigkeit der Abstandsmessung beginnt man am bestem mit einem Praktikumsversuch:

    https://users.physik.fu-berlin.de/~essenber/Dateien/VersucheGP/fap-script.pdf

    Das Limit der Genauigkeit zu bestimmen erfordert quantenmechanische Betrachtungen:

    https://www.phys.lsu.edu/~jdowling/PHYS7353/lectures/Ley87.pdf

  30. #30 Niels
    8. Februar 2016

    Hast du für den E-Modul der Raumzeit eine seriöse Quelle? ich habe da bisher immer nur etwas windige Quellen gefunden und hätte gern mal ne genaue Erklärung, wie diese Zahl zu Stande kommt.

    Siehe Herr Senf, das ist tatsächlich einfach nur die Kopplungskonstante in den Einsteingleichungen.

    Quelle zum Beispiel hier:
    https://www.gravity.uwa.edu.au/docs/review.pdf

    In Newtonian physics spacetime is an infinitely rigid conceptual grid. Gravitational waves
    cannot exist in this theory.
    They would have infinite velocity and infinite energy density because in Newtonian gravitation the metrical stiffness of space is infinite.
    Conversely general relativity introduces a finite coupling coefficient between curvature of spacetime, described by the Einstein curvature tensor, and the stress energy tensor which describes the mass–energy which gives rise to the curvature.
    This coupling is expressed by the Einstein equation
    T = [c^4/8πG]*G. (1.1)

    Here T is the stress energy tensor and G is the Einstein curvature tensor, c is the speed of light and G is Newton’s gravitational constant.
    The coupling coefficient c4/(8πG) is an enormous number, of order 10^43.
    This expresses the extremely high stiffness of space which is the reason that the Newtonian law of gravitation is an excellent approximation in normal circumstances, and why gravitational waves have a small amplitude, even when their energy density is very
    high.
    The existence of gravitational waves is intuitively obvious as soon as one recognizes that spacetime is an elastic medium. The basic properties of gravity waves can be easily deduced from our knowledge of Newtonian gravity, combined with knowledge that spacetime curvature is a consequence of mass distributions.

    Halte ich persönlich übrigens für keine besonders gelungene oder hilfreiche Veranschaulichung für Gravitationswellen.
    Kommt mir auch nicht ganz koscher vor, das Elastizitätsmodul der Mechanik mit der Kopplungskonstante von Materieanteil und Geometrieanteil (Einstein-Hilbert-Wirkung) der ART-Wirkung zu identifizieren.
    Davon abgesehen ist es ja nicht ganz unwesentlich, dass es hier um die Raumzeit geht und eben nicht nur um den Raum, da verliert sich dann sowieso die Vergleichbarkeit.

  31. #31 Niels
    8. Februar 2016

    @MartinB

    Hast du für den E-Modul der Raumzeit eine seriöse Quelle? ich habe da bisher immer nur etwas windige Quellen gefunden und hätte gern mal ne genaue Erklärung, wie diese Zahl zu Stande kommt.

    Siehe Herr Senf, das ist tatsächlich einfach nur die Kopplungskonstante in den Einsteingleichungen.

    Quelle zum Beispiel hier:
    https://www.gravity.uwa.edu.au/docs/review.pdf

    In Newtonian physics spacetime is an infinitely rigid conceptual grid. Gravitational waves
    cannot exist in this theory.
    They would have infinite velocity and infinite energy density because in Newtonian gravitation the metrical stiffness of space is infinite.
    Conversely general relativity introduces a finite coupling coefficient between curvature of spacetime, described by the Einstein curvature tensor, and the stress energy tensor which describes the mass–energy which gives rise to the curvature.
    This coupling is expressed by the Einstein equation
    T = [c^4/8πG]*G. (1.1)

    Here T is the stress energy tensor and G is the Einstein curvature tensor, c is the speed of light and G is Newton’s gravitational constant.
    The coupling coefficient c4/(8πG) is an enormous number, of order 10^43.
    This expresses the extremely high stiffness of space which is the reason that the Newtonian law of gravitation is an excellent approximation in normal circumstances, and why gravitational waves have a small amplitude, even when their energy density is very
    high.
    The existence of gravitational waves is intuitively obvious as soon as one recognizes that spacetime is an elastic medium. The basic properties of gravity waves can be easily deduced from our knowledge of Newtonian gravity, combined with knowledge that spacetime curvature is a consequence of mass distributions.

    Halte ich persönlich übrigens für keine besonders gelungene oder hilfreiche Veranschaulichung für Gravitationswellen.
    Kommt mir auch nicht ganz koscher vor, das Elastizitätsmodul der Mechanik mit der Kopplungskonstante von Materieanteil und Geometrieanteil (Einstein-Hilbert-Wirkung) der ART-Wirkung zu identifizieren.
    Davon abgesehen ist es ja nicht ganz unwesentlich, dass es hier um die Raumzeit geht und eben nicht nur um den Raum, da verliert sich dann sowieso die Vergleichbarkeit.

  32. #32 Herr Senf
    8. Februar 2016

    Hallo beisammen, wir wundern uns bei quanten.de auch:
    nach meinem Überschlag sind es nicht 10^24 GPa, sondern 10^34 GPa.
    Wer hat noch Lust nachzurechnen?

  33. #33 Sven Greffenius
    Berlin
    8. Februar 2016

    Geht man davon aus, dass bei einer Verschmelzung von zwei schwarzen Löchern, Energie nur in Form von Gravitationswellen abgegeben wird?

  34. #34 Karl Mistelberger
    8. Februar 2016

    Keine Lust zum Nachrechnen, aber auch kein Senf: The Strength of Space-Time Elasticity.

  35. #35 Krypto
    8. Februar 2016

    @Sven:
    Wenn man die umgebende Materie vernachlässigt, ja.

  36. #36 Blix
    8. Februar 2016

    Top Zusammenfassung zum denkbar besten Zeitpunkt raus.
    Inklusive: “Es gibt ein Signal und Nobelpreise für alle.”
    Darum gibts von mir 2 Daumen hoch und: https://www.quickmeme.com/meme/3oh3cy

  37. #37 Bernd
    Innsbruck
    8. Februar 2016

    Nach dem BICEP2-Megaflop von vor bald zwei Jahren wird man hoffentlich keine leichtfertigen Schnellschüsse mehr raushauen! Btw: Weiß jemand im Forum, was es in Sachen “Primordiale Gravitationswellen” / “B-Moden im Polarisationsmuster des CMB” Neues gibt? Stehen bald frische Ergebnisse (diesmal um Schmutzeffekte bereinigt) zur Veröffentlichung an? Von BICEP3 und dem Keck Array hört und liest man gar nichts mehr, entsprechende Infos sind auf deren Seiten leider keine zu finden …

  38. #38 Steffen
    9. Februar 2016

    Ein Laie fragt sich: angenommen ein sehr schwerer aber völlig inaktiver (nichts durch andere Vorgänge abstrahlender) Körper wird von was anderem schwerem beschleunigt und gibt dadurch Energie in Form von Gravitationswellen ab: verschwindet der Körper irgendwann, weil ihm die Masse/Energie abhanden kommt?

  39. #39 fherb
    9. Februar 2016

    Ganz sicher gibt es gute Ergebnisse! Sonst wäre ja der ganze Versuchsaufbau völlig falsch bemessen oder ausgerechnet dieses Detail widerspricht Einstein und allem, was auf der Therorie aufbaut!

    Hallo, erst mal!

    @Alderamin u.a.:

    Benötigt man wirklich die Quantenmechanik, um die Auflösung zu bestimmen? Das stell’ ich mir in einem kilometergroßen Messaufbau nicht einfach vor…

    Das Interferometer wird so eingestellt, dass sich die Wellen beider Strahlen gegeneinander “aufheben”. Wie genau man diese Aufhebung bestimmen kann, ist meiner Meinung nach tatsächlich ein quantenmechanisches Problem.

    Das “Aufheben” tun sie aber nur im Mittel: seismische Bewegungen jeden Ursprungs stauchen die Anordnung und das Aufhebungssignal wird ganz erheblich Rauschen. Mit dem Parallelaufbau wird nun eine Korrelation hergestellt. Wenn die Richtung der Welle bekannt ist oder abgeschätzt werden kann, dann lässt diese Korrelation (über die Laufzeit) ein signifikantes Signal entstehen, dass auffällig (also eben signifikant) gegenüber dem Rauschen ist. Und es ist sicher kein Zufall, dass man ein rotierendes Objekt beobachtet: Also etwas, was eine periodische Wellenform sendet, die über längere Zeit stabil ist. Viele Abtastwerte sind gut für die Statistik. – Und ich kann mich nicht mit dem Gedanken anfreunden, dass ein “bisschen Quantenmechanik” reicht, um die Signifikanz des Signals gegenüber des Rauschens zu ermitteln. So einfach ist das ganz bestimmt nicht dahergerechnet.

    Aus meiner Sicht:

    1) Die Kompensationsmethode der beiden Laserstrahlen stellt bezüglich ihrer Wellenlänge ein Maß da, welche Wegänderungen (terrestrisch oder durch Gravitationswellen) messtechnisch grundsätzlich erfassbar ist. (Auflösung des AD-Wandlers des Lichtsignals der beiden überlagerten Laserstrahlen.)

    2) Die örtlichen Gegebenheiten des Aufbaus und des Referenzaufbaus mit den jeweils umgebenden Verkehrsanlagen usw. bestimmen das Rauschverhalten beider Anordnungen.

    3) Aus 1 und 2 ergibt sich auch für das Korrelationsergebnis ein Maß für das Rauschen.

    4) Die Signifikanz und damit Genauigkeit gegenüber dem Grundrauschen ergibt sich nun aus der Messdauer (Zahl der Abtastwerte im Maßstab der zu erwartenden Schwingungsfrequenz des Gravitationsfeldes) und der Voraussetzung, dass über die Messdauer das Signal periodisch ist oder die Änderung der Periodizität als bekannt angenommen werden kann.

    Gerade weil beide Versuchsaufbauten sehr weit voneinander entfernt sind und seismische Ursachen deshalb sehr wenig mit der Laufzeit der Gravitationswellen korrelativ auf das Messergebnis einwirken, verbleibt im Wesentlichen noch das örtliche thermische und seismische Rauschen (als Differenzsignal der beiden Messysteme) aus den Umgebungsbedingungen plus das sämtlicher elektronischer Bauteile als Grenzwert für die Auflösung.

    @ alle:

    Was mich viel mehr interessieren wird (falls die Veröffentlichung das bringt, was gemunkelt wird): Kann tatsächlich dieses Ergebnis auf ein bestimmtes bekanntes kosmisches System zurückgeführt werden, oder ist die Ursache für die Wellen nur eine Theorie? Schwarze Löcher sind ja nun indirekt nachgewiesen. Aber ausgerechnet die Beiden, die für die Messung die Ursache sind? Das kanv ich mir nicht vorstellen.

  40. #40 Herr Senf
    9. Februar 2016

    Es gibt eine “black-box” 26. Sep 2015,
    zwei Mitarbeiter protokollieren ständig, was sich tut.
    Ergebnis G184098 … wird’s das sein?

  41. #41 Karl Mistelberger
    9. Februar 2016

    > fherb, 9. Februar 2016
    > Benötigt man wirklich die Quantenmechanik, um die Auflösung zu bestimmen?

    Ja, die Quantennatur des Lichts bedingt eine inhärente Grenze bei der Messung. Gegen das Schrotrauschen ist letztlich kein Kraut gewachsen. Kohärentes Licht kann die Situation etwas verbessern: For example phase-squeezed light can improve the phase read out of interferometric measurements (see for example gravitational waves).

  42. #42 black ice
    9. Februar 2016

    Coole Sache das.

    Wieder einmal ein informativer Beitrag. Bin schon gespannt auf morgen!

    Meine einfache Antwort auf die Frage nach dem warum ist “weil wir es können”. Die Neugierde sitzt tief in der menschlichen Natur, und bis jetzt hat es immer interessante Spin-offs nach großartigen Entdeckungen gegeben.

    Kann man Teleskope bauen, die Gravitationswellen für Beobachtungen verwenden, so wie beielektromagnetischen?

  43. #43 Jockel
    9. Februar 2016

    Im DLF von gestern hieß es: Bislang aber ist noch nicht einmal bestätigt, dass am Donnerstag tatsächlich eine Pressekonferenz stattfinden wird.

    Alle Gerüchte sollen auf “ausschmückende” Berichte in Science beruhen.

  44. #44 Bullet
    9. Februar 2016

    @Steffen:

    angenommen ein sehr schwerer aber völlig inaktiver […] Körper wird von was anderem schwerem beschleunigt und gibt dadurch Energie in Form von Gravitationswellen ab: verschwindet der Körper irgendwann, weil ihm die Masse/Energie abhanden kommt?

    Nehmen wir elektromagnetische Energie statt Gravitationswellen. Der Körper strahlt jetzt Photonen ab. Deine Frage würde in diesem Fall lauten: “Wenn so ein Körper glüht, verschwindet er irgendwann einfach, weil ihm die Masse/Energie abhanden kommt?”
    Die Antwort ist jetzt offensichtlich: natürlich nicht. Das Glühen wird nur immer schwächer, weil der Körper abkühlt. Irgendwann ist er so kalt wie der umgebende Raum. (Okay, okay, das kann, äh, etwas dauern…)
    Auf die Gravitationswellen bezogen, heißt das, daß er natürlich nicht verschwindet. Aber er würde natürlich Masse verlieren und damit weniger Gravitationswellen erzeugen, die dann weniger Energie davontragen usw… Das Strahlungsgesetz sollte eigentlich im Prinzip für G-Wellen genauso wie für EM-Wellen gelten.

    Was mich jetzt zu der Frage bringt: LIGO sieht mir erstmal so aus wie ein Detektor, der auf Amplituden optimiert ist. Aber was ist mit der Frequenz der Gravitationswellen? Wie hoch ist eigentlich die spektrale Auflösung dieses Gerätes?
    Anyone?

    (An dieser Stelle übrigens nochmal einen Dank an Fellianer (#12), der nachgesehen hat, wie das mit der horizontalen Exaktesse war.)

  45. #45 Alderamin
    9. Februar 2016

    @Steffen, Bullet

    Wenn die Erde die Sonne umkreist, oder sich zwei Neutronensterne umkreisen, dann geht dabei nur Bewegungs- bzw. Lageenergie (potenzielle Energie) verloren, keine Masseenergie. Da verschwindet gar nichts an Masse, die abgestrahlten Gravitationswellen wirken nur bremsend, so ähnlich wie die Induktion beim Elektromagnetismus (z.B. Wirbelstrombremse), so weit ich das richtig verstehe.

    Die zwei umeinander kreisenden Schwarzen Löcher waren mit mächtig Geschwindigkeit, fast Lichtgeschwindigkeit am Ende, unterwegs, ich nehme an, bei der verlorenen Masse handelte es sich dort ebenfalls um Bewegungsenergie, die per relativistischer Massenzunahme zur Masse der Schwarzen Löcher beitrug, bevor sie verschmolzen. Und das waren dann halt 3 Sonnenmassen zu dem Zeitpunkt, an dem die Einzelmassen der Schwarzen Löcher bestimmt wurden.

    Das ist aber nur meine Annahme, mit sowas kenne ich mich überhaupt nicht aus. Wenn die Veröffentlichung draußen ist, wird die Frage nach dem Ursprung der drei Sonnenmassen sicherlich häufiger gestellt werden und irgendwer (z.B. die Autoren) eine kundige Antwort geben.

  46. #46 Florian Freistetter
    9. Februar 2016

    @Jockel:“Bislang aber ist noch nicht einmal bestätigt, dass am Donnerstag tatsächlich eine Pressekonferenz stattfinden wird.”

    Diese Pressekonferenz wurde mittlerweile offiziell von LIGO angekündigt: https://www.ligo.org/news/media-advisory.php

  47. #47 Alderamin
    9. Februar 2016

    @fherb

    So wie Du die Funktionsweise von LIGO beschreibst, stelle ich mir das auch vor, aber wissen weiß ich’s nicht.

    : Kann tatsächlich dieses Ergebnis auf ein bestimmtes bekanntes kosmisches System zurückgeführt werden, oder ist die Ursache für die Wellen nur eine Theorie? Schwarze Löcher sind ja nun indirekt nachgewiesen. Aber ausgerechnet die Beiden, die für die Messung die Ursache sind? Das kanv ich mir nicht vorstellen.

    Man kann ausrechnen, welche Prozesse welche Wellen (Frequenz, Intensität, Amplituden-Hüllkurve) erzeugen, und es sind bei LIGO Einzelereignisse wie verschmelzende Scharze Löcher oder Neutronensterne, die im Nachweisbereich des Instruments liegen. Man kann das vielleicht vergleichen mit der Wahrnehmung des Lichts einer Supernova im Optischen, die sticht ja auch aus der Masse der Sterne einer Galaxie heraus.

    Hier wurde halt beobachtet, wie die Periode der beiden kreisenden Objekte immer mehr zunahm, bis zur Verschmelzung, und das resultierende Schwarze Loch dann auf “Kerr-Form” relaxte. Es gibt ja zwei wichtige Lösungen der ART-Gleichungen für Schwarze Löcher, die Schwarzschild-Lösung für nichtrotierende mit einer Punkt-Singularität und die Kerr-Lösung für rotierende mit einer ringförmigen Singularität und mitrotierender Raumzeit, wobei hier nur die Kerr-Lösung in Frage kommt. Die Kerr-Lösung beschreibt aber nur den stabilen Endzustand, der bei der Verschmelzung zunächst noch nicht erreicht war. Der Übergang von der Hantelform zweier verschmelzender Schwarzer Löcher zur ringförmigen Singularität ist das, was man am Ende beim Ausklingen der Schwingung beobachtet hat (“ringdown to Kerr”). Man hat also ein sehr singuläres Ereignis beobachtet.

    Nun hat LIGO nur zwei Messstationen. Damit kann man leider keine Richtung angeben, wo die Quelle der Gravitationswellen zu verorten ist. Man bräuchte wenigstens eine dritte Station, um den Ort einigermaßen anzupeilen. Die Frage wäre dann aber, ob man dort optisch oder als Gammaburst überhaupt etwas gesehen hätte. Wenn die Schwarzen Löcher Akkretionsscheiben gehabt hätten, dann wäre sicherlich solche Strahlung entstanden, andernfalls wohl nicht. Gammabursts sehen wir aber gemeinhin auch nur, wenn wir auf der Achse des Jets liegen, die senkrecht zur Akkretionsscheibe ausgestoßen werden, und das ist im allgemeinen ja nicht der Fall. Wäre natürlich cool, wenn man das konkrete Ereignis auch anderweitig beboachten könnte, das gäbe zusätzliche Sicherheit (wie bei Supernova 1987A und ihren Neutrinos, oder Gamma-Bursts und optisch sichtbaren Supernovae).

  48. #48 ZauberDerSterne.de
    9. Februar 2016

    @Jockel:

    Da hast du Recht, allerdings wurden am Montagabend gleich 4(!) zeitgleiche Pressekonferenzen für Donnerstag um 16:30 MEZ angekündigt.

  49. #49 noonscoomo
    Berlin
    9. Februar 2016

    Ist es nicht so, dass eine Gravitationswelle nicht nur den Raum, sondern die Raumzeit “verbiegt”. Und sollte das nicht auch dazu führen, dass das Licht zwar z.B. einen längeren Weg zurücklegen muss weil der Raum gedehnt ist, dafür aber auch mehr Zeit hat, weil die Zeit eben auch gedehnt ist. Und sollte sich das dann nicht gegenseitig aufheben?

  50. #50 Bullet
    9. Februar 2016

    @Alderamin:

    Die Frage wäre dann aber, ob man dort optisch oder als Gammaburst überhaupt etwas gesehen hätte.

    Naja … ich persönlich würde ja eher nach einem Radioblitz suchen. Weißt schon: Gammaburst -> Gravitationsloch -> Dilatation … oder?

  51. #51 Jockel
    9. Februar 2016

    Ist es nicht so, dass eine Gravitationswelle nicht nur den Raum, sondern die Raumzeit “verbiegt”. Und sollte das nicht auch dazu führen, dass das Licht zwar z.B. einen längeren Weg zurücklegen muss weil der Raum gedehnt ist, dafür aber auch mehr Zeit hat, weil die Zeit eben auch gedehnt ist. Und sollte sich das dann nicht gegenseitig aufheben?

    Ich stellte mir das laienhaft bisher auch so vor, dass die Lichtgeschwindigkeit in jedem Raum gleich schnell ist. In einem sich ausdehnenden Raum (also die eine Richtung) wird das dann langwelliger (“Rotverschiebung”), in einem sich zusammenziehenden Raum (die andere Richtung) kurzwelliger. Und dass dieser Unterschied durch Überlagerung der Frequenzen gemessen würde. Aber wenn die sagen, dass das einfach nur die Laufzeit ist, dann werden die schon recht haben.

  52. #52 Alderamin
    9. Februar 2016

    @noonscoomo, Jockel

    Siehe #23 und den dortigen Link.

    @Bullet

    Bei einer Hypernova oder zwei verschmelzenden Neutronensternen fällt Materie auch in ein Schwarzes Loch und es gibt einen Gammaburst, keinen Radioburst. Der Burst entsteht im Jet (Bremsstrahlung, glaube ich), etwas entfernt vom Ereignishorizont. Und er geht rechtwinklig zur Drehung der Akkretionsscheibe ab. Daher gibt’s da nicht viel Dilatation.

  53. #53 Alderamin
    9. Februar 2016

    @myself

    Hier wurde halt beobachtet, wie die Periode der beiden kreisenden Objekte immer mehr zunahm, bis zur Verschmelzung

    Argh, “abnahm”. Zunehmen tat die Frequenz.

  54. #54 Artur57
    9. Februar 2016

    @Bullet

    Hier ist es eher so, daß eine Gravitationswelle eine Lichtwelle erst staucht und dann wieder streckt. Im Ergebnis ist dann aber spätestens die Phasenlage eine andere. Und genau dazu ist dieser Versuchsaufbau bei LIGO gedacht.

    Das wäre einleuchtend, allerdings erklärt es Wikipedia nicht so :

    Durchquert eine Gravitationswelle das Observatorium, ändern sich die relativen Längen der Arme des Interferometers. Während der eine Arm gedehnt wird, verkürzt sich der andere Arm. Das ruft eine Phasenverschiebung der beiden Teilwellen des Laserlichtes hervor und deren Interferenz ändert die Intensität des gemessenen Lichtes

    Also demnach werden die Arme tatsächlich kürzer und länger und daraus resultieren dann verschiedene Phasenlagen.

  55. #55 Higgs-Teilchen
    Im Standardmodell oben rechts
    9. Februar 2016

    “Für diesen ersten indirekten Nachweis haben die beiden immerhin 1993 den Nobelpreis für Physik bekommen.”

    Schon 1993? Krass, wusste ich auch noch nicht. War ja dann noch vor dem ersten nachgewiesenen Exoplaneten. :-O Staun……

  56. #56 Artur57
    9. Februar 2016

    Leider ist bei der Schwingungskurve keine Zeit angegeben, das PDF sagt allerdings, dass der Beobachtungszeitraun drei Jahre beträgt (soweit Theorie).

    Wenn wir das nun als Größenordnung nehmen, sowie beim realen Objekt die Beobachtung, dass seine Masse um drei Sonnenmassen abgenommen hat, dann hieße das, dass hier pro Jahr eine Sonnenmasse in Gravitationswellen umgesetzt wird. Das wäre aber krass, das hieße, dass solche Objekte die stärksten Energiequellen des Universums sind.

    Die bislang führenden Blauen Riesen sind bis zu 250 Sonnenmassen schwer und verbrennen diese teilweise, bis sie nach einigen -zig Millionen Jahren in einer Supernova enden. Also deutlich weniger Massenverbrauch und Energieabgabe, als diese mergernden Schwarzen Löcher.

    Ich dachte immer, das liege so etwa im 50-Watt-Bereich. Da ist Umdenken angesagt.

  57. #57 Higgs-Teilchen
    Im Standardmodell oben rechts
    9. Februar 2016

    Ach ja, noch zwei Fragen an alle, die es wissen:

    “Sie würden einander immer näher kommen, miteinander kollidieren und schließlich zu einem einzigen schwarzen Loch verschmelzen.”

    Wie lange dauert so was? Also von Annäherung bis endgültiger Verschmelzung?

    Und:

    “Je näher das schwarze Loch an der energetisch günstigen Form ist, desto schwächer werden die Wellen, bis sie schließlich ganz verschwinden.”

    Wie lange gibt es dann die Wellen noch ab? Von welchen Zeiträumen geht man aus?

    Danke schon mal.

    Lg H.

  58. #58 noonscoomo
    Berlin
    9. Februar 2016

    @Alderamin
    Den Hinweis in #23 hatte ich schon verstanden (glaub ich zumindest), aber m.E. ist es nicht das Gleiche, ob die Wellenlänge verändert wird oder die Zeitbasis. Oder anders gesagt, das hängt vom Betrachter ab. Die Gravitationswelle verändert ja nicht nur die Laufzeit und die Wellenlänge, sondern eben auch den zeitlichen Massstab. Ich mache ein Beispiel, ein Laserimpuls fliegt durch Arm A und B während eine Gravitationswelle so daherkommt, dass der Arm A etwas länger wird, B aber nicht. Aber dort wo er länger wird, vergeht auch die Zeit etwas langsamer. Von einem unabhängigen Beobachter aus könnte man sagen, die Wellenlänge hat sich geändert. Aber es kann ja keinen unabhängigen Beobachter geben, denn dazu müsste Information ausgetauscht werden und das geht nur mit Lichtgeschwindigkeit und die Information müsste auch erst durch den Bereich vergrösserten Raumes bei langsamerer Zeit. Kurzum, wenn ich dort stehe, wo sich die Pulse aus A und B wieder treffen, kommen sie gleichzeitig an, da A zwar einen grösseren Weg zurückgelegt hat, dafür aber auch mehr Zeit zur Verfügung hatte. D.h. gerade bei Betrachtung der “time of flight” hätte ich ein Problem, die Gravitationswelle nachzuweisen. Ich bin gespannt, was da übermorgen veröffentlicht wird.

  59. #59 Alderamin
    9. Februar 2016

    @Artur57

    Wenn wir das nun als Größenordnung nehmen, sowie beim realen Objekt die Beobachtung, dass seine Masse um drei Sonnenmassen abgenommen hat, dann hieße das, dass hier pro Jahr eine Sonnenmasse in Gravitationswellen umgesetzt wird. Das wäre aber krass, das hieße, dass solche Objekte die stärksten Energiequellen des Universums sind.

    Es ist noch ein bisschen krasser:

    Bei Luboš Motl ist eine ähnliche Kurve abgebildet, und da gibt’s auch eine Zeitachse. Es handelt sich um ca. 0,4 s im Graphen, keine 3 Jahre… Jede Wellenlänge ist ein Umlauf, und die beiden Objekte rasen mit fast Lichtgeschwindigkeit umeinander herum, bevor sie verschmelzen.

    Allerdings werden die drei Sonnenmassen nicht in dieser kurzen Zeit verbraten. Im Artikel schreibt Motl etwas von “einigen Minuten”. Er rechnet dann mit 100 s, um eine Leistung von 10^46 W abzuschätzen.

  60. #60 Alderamin
    9. Februar 2016

    @Higgs-Teilchen

    Deine Frage sollte mit dem oben verlinkten Graphen beantwortet sein.

    @noonscoomo

    ein Laserimpuls fliegt durch Arm A und B während eine Gravitationswelle so daherkommt, dass der Arm A etwas länger wird, B aber nicht. Aber dort wo er länger wird, vergeht auch die Zeit etwas langsamer.

    Der Witz an der SRT ist, dass das Licht von einem ruhenden Beobachter auch dort mit Lichtgeschwindigkeit unterwegs zu sein scheint, wo die Zeit langsamer läuft, Beispiel Lichtuhr: sowohl der ruhende Beobachter, als auch der mit der Uhr bewegte sehen den Lichtstrahl gleich schnell.

    Bei der ART ist das allerdings nicht der Fall, da liegt Beschleunigung vor (man sieht eine Lichtuhr in einem Schwerefeld von außerhalb langsamer ticken).

    Auf der LIGO-Seite wird jedenfalls mit der veränderten Strecke argumentiert, scheint also eher die SRT zu greifen. Ich weiß auch ich nicht mehr, als da steht, ausrechnen kann ich so was nicht.

  61. #61 Bernd
    9. Februar 2016

    Letztlich wäre (!) diese Entdeckung eine furiose finale Letztbestätigung der ART, vergleichbar der Detektion des Higgs-Teilchens 2012, das ja auch “nur” die bereits etablierte Standartheorie experimentell bestätigt hat. Frage: Hätte die Entdeckung der Gravitationswellen auch Relevanz für noch unbeantwortete Fragen der Kosmologie, also über die ART hinausweisend?

  62. #62 noonscoomo
    Berlin
    9. Februar 2016

    @Alderamin

    Auf der LIGO-Seite wird jedenfalls mit der veränderten Strecke argumentiert, scheint also eher die SRT zu greifen.

    Na genau das ist ja mein Problem, das ist m.E. nicht zulässig, denn ich habe es ja hier mit einem quasi beschleunigten System zu tun, wenn da eine Graviationswelle über mich rüber schwappt. Letztlich geht es doch eben darum eine Vorhersage der ART zu belegen.

  63. #63 Alderamin
    9. Februar 2016

    @Bernd

    Hätte die Entdeckung der Gravitationswellen auch Relevanz für noch unbeantwortete Fragen der Kosmologie, also über die ART hinausweisend?

    Na klar, z.B. als experimentelle Bestätigung der Modell, die wir uns von Schwarzen Löchern machen, denn gesehen hat noch niemand eines, nach bisherigen Beobachtungen ist das einfach unsichtbare, kompakte Masse, die gelegentlich von einer leuchtenden Materiescheibe mit Jet umkreist wird. Alles darüber hinaus Gehende ist Theorie.

    Ähnliches gilt für verschmelzende Neutronensterne oder den Kernkollaps einer Supernova.

    Wenn man, wie bei BICEP2 voreilig gemeldet, die Muster von Gravitationswellen in der Hintergrundstrahlung fände, wäre das eine Bestätigung der Inflationstheorie und könnte verschiedene Modellvarianten derselben bestätigen bzw. ausschließen. Aber dieser Punkt ist weiterhin offen, dazu kann LIGO nichts beitragen.

    Gravitationswellen öffnen ein neues Fenster zum Weltall, so wie früher Radiowellen oder Röntgen- und Gammastrahlung. Damit wird man auch neue Erkenntnisse gewinnen.

  64. #64 noonscoomo
    Berlin
    9. Februar 2016

    @Artur57

    Also demnach werden die Arme tatsächlich kürzer und länger und daraus resultieren dann verschiedene Phasenlagen.

    Aber kürzer oder länger bezieht sich doch auf meinen Maßstab und der ändert sich durch die Gravitationswelle ja gleich mit. Denn mein Massstab ist doch Weg pro Zeit. Nehmen wir mal an, das wäre keine Gravitationswelle, sondern ganz statisch, wir legen die beiden Achsen flach auf den Boden, messen die Laufzeit der beiden Laserimpulse und bestimmen damit die Länge der beiden Arme. Da fängt’s schon an problematisch zu werden, denn dabei muss ich erst mal annehmen, dass die Lichtgeschwindigkeit in beiden Richtungen identisch ist. So 100% klar ist schon das nicht mal. Aber egal, das Detail ignorieren wir kurz und nehmen an dass das so ist. Wir stellen jetzt also fest, dass beide Arme gleich lang sind, nun rotiere ich das Konstrukt in Gedanken um 90° so, dass der Arm B senkrecht in die Luft zeigt, dann ist also da weiter oben weniger Gravitation, richtig? Also wird der Arm B etwas länger. Gleichzeitig geht da oben aber auch die Zeit anders als unten am Boden. Eigentlich müsste sie ja schneller gehen. D.h. das Licht hätte weniger Zeit… Oh, hm. Dadurch würde das Licht ja eher noch länger brauchen, bis es wieder zurück ist… äh. Ok. Ist das so? Dann wäre ja alles gut und die veränderte Zeit würde den Effekt sogar verstärken. pew. Knoten im Hirn.

  65. #65 Alderamin
    9. Februar 2016

    @noonsoomo

    nun rotiere ich das Konstrukt in Gedanken um 90° so, dass der Arm B senkrecht in die Luft zeigt, dann ist also da weiter oben weniger Gravitation, richtig?

    Nö, falsch (mal abgesehen davon, dass das Gewicht die Konstruktion zusammendrückt). Die Zeit läuft im Schwerefeld zwar langsamer, Längen bleiben hingegen unverändert (bis auf die gekrümmte Geometrie, die aber erst auf größeren Abständen eine Rolle spielt).

  66. #66 Alderamin
    9. Februar 2016

    @noonscoomo

    Oben im Zitat fehlte noch

    Also wird der Arm B etwas länger

    denn natürlich herrscht oben weniger Schwerkraft.

    Es gibt aber doch eine Formel für die Längenverkürzung im Gravitationsfeld, wie ich eben gefunden habe, kannte ich nicht: L =L₀*(1-GM/(Rc²)).

    Das ist aber ein anderer Faktor als bei der Zeitdilatation durch Gravitation: t=t₀/√(1-2GM/(Rc²)). Das kann sich also nicht aufheben.

  67. #67 Noonscoomo
    Berlin
    9. Februar 2016

    @Alderamin
    Cool, der erste Link ist sehr hilfreich. Da ist ja alles kompakt einmal beieinander. Das guck ich mir in Ruhe mal an. Tausend Dank

  68. #68 DasKleineTeilchen
    terra
    9. Februar 2016

    @Bullet, No.03:

    sehr interessantes interview mit nem flat-earther:

    https://www.youtube.com/watch?v=hsOz_J6tJVU

    kurz gesagt; diese leute haben schlicht einen heidenschiss davor, daß die erde eben *nicht* der mittelpunkt des universums ist. die “argumentation” läuft auch hier im interview nicht anders (kurzer, erhellender moment des verquatschens), tenor: “if the eart were like the standard-model, it would be total insignificant, just like other planets in the universe…; das sagt dieser flat-earther allen ernstens und ich bin mir ziemlich sicher, daß das bei fast allen dieser leute *die* treibende kraft dahinter ist, an solch einem bullshit festzuhalten.

  69. #69 Artur57
    10. Februar 2016

    @noonscoomo

    Wo ich dieses heiße Eisen eigentlich liegen lassen wollte.

    Also meiner Meinung nach wird das Geschehen am Interferenzpunkt ausschließlich durch die durchlaufenen Strecken und die Wellenlänge des Lichts bestimmt. Die Geschwindigkeit des Lichts ist völlig unerheblich. So ist es auch einleuchtend.

    Was die LIGO-Betreiber anders sehen. Denn, wie Alderamin zitiert:

    Die Frage ist ziemlich gut. Die LIGO-Seite hat die Antwort: es kommt nicht auf die Wellenlänge oder die Zahl der Wellen in den Interferometerarmen an, sondern auf die Laufzeit, und die ist wegen der Konstanz der Lichtgeschwindigkeit nur von der Länge der Arme abhängig.

    Was passiert, wenn wir die Anordnung drehen, ist damit nicht gesagt. Arm B wird länge (wenn wir sein Gewicht ignorieren). Aber aucch das Licht ändert seine Wellenlänge und auf dem Rückweg wieder zurück. Wird schwer.

  70. #70 noonscoomo
    Berlin
    10. Februar 2016

    @Artur57

    Aber aucch das Licht ändert seine Wellenlänge und auf dem Rückweg wieder zurück. Wird schwer.

    Ja, genau das war ja meine denke. Ich fürchte da kommen wir um’s ausrechnen nicht drumrum. Alderamin hat ja das nötige Handwerkszeug geliefert. Ich hatte halt gehofft, dass sich schon mal jemand mit dieser Frage beschäftigt hat. Aber ich konnte bisher auch nix dazu finden. Irgendwie erinnert mich das ein wenig an Prof. David Wiltshire, der mit einer ganz ähnlichen Betrachtungsweise die dunkle Energie zu erklären versucht: https://www2.phys.canterbury.ac.nz/%7Edlw24/universe/

  71. #71 Jockel
    10. Februar 2016

    Interessant finde ich in diesem Zusammenhang das Einsteinteleskop (ich konnte nicht finden, ob der Astrodicticum Simplex darüber schonmal gebloggt hat).

    Die Forscher gehen davon aus, dass das etwa 1 Mrd. Euro kosten würde und sie die Mittel für den Bau und Betrieb nur einwerben können, wenn LIGO (WA und LA/USA) und Virgo (Toskana/IT) wenigstens ab und zu mal Gravitationswellen nachweisen würden.

    LIGO hat 2015 ein Upgrade erhalten, wodurch die Empfindlichkeit um das 7-10fache gesteigert werden sollte. Virgo bekommt das momentan und wird es dieses Jahr abschließen. Beides bezeichnen die Gravitationswellenforscher als Gravitationsteleskope der 2. Generation. Das Einsteinteleskop soll das auf das 100fache steigern und die 3. Generation darstellen.

  72. #72 Karl Mistelberger
    10. Februar 2016

    > Artur57, 10. Februar 2016
    > Wo ich dieses heiße Eisen eigentlich liegen lassen wollte. Also meiner Meinung nach wird das Geschehen am Interferenzpunkt ausschließlich durch die durchlaufenen Strecken und die Wellenlänge des Lichts bestimmt. Die Geschwindigkeit des Lichts ist völlig unerheblich. So ist es auch einleuchtend.

    > Was die LIGO-Betreiber anders sehen. Denn, wie Alderamin zitiert: Die Frage ist ziemlich gut. Die LIGO-Seite hat die Antwort: es kommt nicht auf die Wellenlänge oder die Zahl der Wellen in den Interferometerarmen an, sondern auf die Laufzeit, und die ist wegen der Konstanz der Lichtgeschwindigkeit nur von der Länge der Arme abhängig.

    > Was passiert, wenn wir die Anordnung drehen, ist damit nicht gesagt. Arm B wird länger (wenn wir sein Gewicht ignorieren). Aber auch das Licht ändert seine Wellenlänge und auf dem Rückweg wieder zurück. Wird schwer.

    Physiker sind durch die Bank einfacher gestrickt: Die Lichtgeschwindigkeit ist konstant. Auf dieser Eigenschaft beruht auch die Definition des Meters. In der Sekunde legt das Licht genau 299 792 458 Meter zurück, ohne Fehlergrenzen und anderes Wenn und Aber sondern per definitionem. Wenn das Licht in dem einen Arm des Interferometers länger oder kürzer unterwegs ist liegt es daran, dass sich die Länge des Arm geändert hat.

  73. #73 noonscoomo
    Berlin
    10. Februar 2016

    @Karl Mistelberger

    In der Sekunde legt das Licht genau 299 792 458 Meter zurück

    Naja, ja, aber wenn sowohl die Sekunde als auch der Meter relativ sind…
    Man nutzt für diese Masseinheiten halt eben Messlatten, die sich mit der Gravitationswelle selbst ändern.
    Wenn du selbst mit samt deinem Zollstock kleiner und wieder grösser wirst, während die Welle vorbeikommt wird dein Meter immer noch ein Meter sein. Gleiches gilt ja auch für deine Uhr.

  74. #74 PDP10
    10. Februar 2016

    @noonscoomo:

    Man nutzt für diese Masseinheiten halt eben Messlatten, die sich mit der Gravitationswelle selbst ändern.
    Wenn du selbst mit samt deinem Zollstock kleiner und wieder grösser wirst, während die Welle vorbeikommt wird dein Meter immer noch ein Meter sein. Gleiches gilt ja auch für deine Uhr.

    Nein, eben nicht.

    Stell dir die Gravitationswelle als ebene Welle vor, die genau in die Richtung des einen Arms des Interferometers einläuft, dh mit der Wellenebene genau senkrecht dazu.

    Die Welle wird diesen Arm “zusammenstauchen”. Den anderen, der genau parallel zur Ebene der Welle ist aber nicht.

    Tadaa.

  75. #75 noonscoomo
    Berlin
    10. Februar 2016

    @PDP10
    Ja, schon klar, aber darum ging ja die ganze Diskussion der letzten Beiträge, dass es nicht 100% klar ist, wie sich Längenkontraktion und Zeitdilatation durch die Welle denn gegenseitig beeinflussen wie in #58 dargestellt.

  76. #76 noonscoomo
    Berlin
    10. Februar 2016

    Was mich gerade noch zusätzlich irritiert, wenn ich das richtig verstanden habe sollte nach dem “merger” der “ringdown”kommen. Aber das würde ja doch bedeuten, dass, und so schaut das auf dem Bild ja auch aus, die beiden SLs zu einem einzigen SL werden. Jetzt hab ich aber in anderen Beiträgen hier auf Astrodicticum Simplex gelesen, dass es von uns aus gesehen unendlich lange dauert, bis etwas durch den Schwarzschildradius durch kommt. Das gilt dann ja bitte auch für ein anderes schwarzes Loch. Wie kann es dann sein, dass wir in der Messung sehen, die beiden SLs zu einem einzigen werden?

  77. #77 Herr Senf
    10. Februar 2016

    Das ist für einen abstrakten entfernten Beobachter so, aber nur bei Kleinigkeiten.
    Für massive Objekte, die selbst die Raumzeit verformen, gilt das so nicht.
    Der Ereignishorizont ist in dem Sinn keine Grenze, ein mitfallender Beobachter kommt durch, ohne was zu merken. Es vereinigen sich zwei “Raumblasen” bei Berührung.
    Wir sehen einfach nur, wie ein neuer Horizont entsteht, reingucken können wir nicht.

  78. #78 Niels
    11. Februar 2016

    @noonscoomo @Alderamin

    Auf der LIGO-Seite wird jedenfalls mit der veränderten Strecke argumentiert, scheint also eher die SRT zu greifen. Ich weiß auch ich nicht mehr, als da steht, ausrechnen kann ich so was nicht.

    Ist es nicht so, dass eine Gravitationswelle nicht nur den Raum, sondern die Raumzeit “verbiegt”.

    Die Gravitationswelle verändert ja nicht nur die Laufzeit und die Wellenlänge, sondern eben auch den zeitlichen Massstab. Ich mache ein Beispiel, ein Laserimpuls fliegt durch Arm A und B während eine Gravitationswelle so daherkommt, dass der Arm A etwas länger wird, B aber nicht. Aber dort wo er länger wird, vergeht auch die Zeit etwas langsamer.

    Eine Gravitationswelle im Gravitationsfeld der Erde “verbiegt” in äußerst guter Näherung tatsächlich nur den Raum, und auch diesen in ganz spezieller Weise.
    Das liegt daran, dass das Gravitationsfeld der Erde vergleichsweise extrem ist. (Deswegen kommen wir ja auch mit der klassischen Mechanik bzw. sogar bei Teilchenbeschleunigern mit der SRT aus.)

    Dadurch wird es möglich, die Nichtlinearität der Einsteinschen Feldgleichungen zu vernachlässigen.
    Man nimmt also an, dass man die Gravitationswelle als ganz spezielle schwache Störung einer Hintergrundraumzeit beschreiben kann.
    Das bedeutet, dass man die Metrik der Gesamt-Raumzeit als Metrik der von der Erde erzeugten Raumzeit (also Schwarzschild) plus der Metrik einer Gravitationswelle (im Vakuum) zu beschreiben.

    Im Allgemeinen wäre das in der ART völlig falsch. Nichtlinearität bedeutet ja gerade, dass man die Gesamtlösung eben nicht einfach linear als Summe der Teillösungen erhält.

    Wenn man aber linearisieren kann, betrachten wir einfach die Lösung der Einsteingleichungen für Gravitationswellen im Vakuum und addieren diese schlicht zur Lösung der Einsteingleichungen für das von der Erde erzeugte Gravitationsfeld.

    Die linearisierten Einsteingleichungen für Gravitationswellen im Vakuum entsprechen exakt der kovarianten Formulierungen der homogenen Maxwellgleichungen, haben also auch die selbe Lösung.
    Gravitationswellen im Vakuum sind also (in linearer Näherung) wie elektromagnetische Wellen im Vakuum transversale Wellen, die sich mit Lichtgeschwindigkeit ausbreiten und nur zwei linear unabhängige Polarisationszustände besitzen.

    In der ART kann man ja beliebige Koordinatensysteme wählen, das entspricht einer Eichung in der Elektrodynamik. (Die Eichfreiheit der ART drückt sich genau dadurch aus, dass man beliebige Koordinatensysteme wählen kann.)

    So wie bei elektromagnetischen Wellen passend eichen kann, damit diese in die räumliche y- und z-Richtungen schwingen
    https://tinyurl.com/zo5f34g
    so eicht man bei Gravitationswellen (d.h. hier eben, dass man ein geeignetes Koordinatensystem verwendet) sinnvollerweise so, dass die beiden Richtungen gerade den rechtwinklig stehenden beiden Armen des Interferometers entspricht.
    https://tinyurl.com/z45djec
    (Wenn man mit Hilfe der Arme ein Koordinatensystem konstruiert, also entlang des einen Armes die räumliche y-Richtung legt und entlang des Anderen die z-Richtung, breitet sich die Welle hier in die z-Richtung aus, also in die Bildebene hinein.

    In linearer Näherung bewirkt eine Gravitationswelle also periodisch gleichzeitig eine Streckung des einen Armes und eine Verkürzung des anderen Armes und umgekehrt.
    Im Bild heißt eine Schwingungsperiode der Welle T_Gw. Würde man die Längenänderungen der beiden Arme in ein Schaubild aufmalen, bekäme man für den einen Arm eine Sinuskurve sin[x] und für den anderen
    Arm eine -sin[x] Kurve.

    Der Unterschied zwischen elektromagnetischer Welle und Graviationswelle in linearer Näherung beruht darauf, dass solche Gravitationswellen Helizität plus/minus 2 besitzen und elektromagnetische Wellen Helizität plus/minus 1.
    Das bedeutet, dass man die eine unabhängige Polarisation einer elektromagnetischen Welle aus der Anderen bekommt, wenn man eine Drehung um 90 Grad um die Ausbreitungsrichtung durchführt.
    Bei Gravitationswellen dagegen bei einer Drehung um 45 Grad.

    Wohlgemerkt, dass gilt alles nur, weil das Gravitationsfeld der Erde schwach genug ist, dass man linearisiern kann.
    Würde man das Interferometer auf einem Neutronenstern aufstellen, müsste man tatsächlich zusätzlich zeitliche Verzerrungen durch die Welle berücksichtigen.

    In einer solchen Raumzeit sind Gravitationswellen auch nicht mehr einfach mehr schöne transversale ebene Wellen mit 2 unabhängigen Polarisationszuständen.
    Im Allgemeinen kann es 10 unabhängige Polarisationszustände geben, die Transversalität kann man also auch vergessen. Tatsächlich ist es eigentlich sogar sinnlos, von Ausbreitung mit Lichtgeschwindigkeit zu sprechen, das gilt für jeden Beobachter nur noch lokal genau an seiner eigenen Position.
    Bei der linearen Näherung ist die Ausbreitungsgeschwindigkeit der Welle ja die Lichtgeschwindigkeit der Hintergrundraumzeit, bei Nichtlinearität verändert die Welle selbst aber eben nichtlokal diese Größe.

    Da helfen also nette Formeln für die Schwarzschildlösung überhaupt nicht weiter. Man müsste für jede spezielle Gravitationswelle die nichtlinearen Einsteingleichungen in ihrer ganzen Schönheit lösen. Da gibt es dann aber keine bekannten allgemeinen analytischen Lösungen mehr, vielmehr müsste man seinen Supercomputer anschmeißen und das Ganze numerisch durchprügeln.

    Ich hab mich bemüht, dass Ganze relativ einfach zu formulieren, ein bisschen Vorwissen über ebene Wellen und Schwindungen musste ich aber voraussetzen.
    Wenn etwas unklar ist, bitte einfach nachfragen.

  79. #79 Niels
    11. Februar 2016

    @noonscoomo @Alderamin

    Ein ziemlicher länger Text mit einer Antwort zu eurem Problem hängt in der Moderation.
    Bei Gelegenheit einfach mal schauen, ob #78 aufgetaucht ist. 😉

  80. #80 walter
    11. Februar 2016

    Kann ich mir eine Gravitationswelle als “Erschütterung” der Gravitation vorstellen und nicht als klassische Welle im Sinne einer Wasserwelle?

    Für mich wäre das logisch.

  81. #81 Franz
    11. Februar 2016

    Scheinbar ist echt was dran. Auf einer meiner Favorites:https://apod.nasa.gov/apod/astropix.html
    steht gerade: Wir warten mit dem heutigen Bild auf die Pressekonferenz um 11:00 von LIGO 🙂

  82. […] Gravitationswellen sind und wie man versucht, sie nachzuweisen, habe ich ja kürzlich schon ausführlich erklärt (und wer noch tiefergehende Informationen sucht, den möchte ich auf diesen Artikel von Markus […]

  83. #83 Florian Freistetter
    11. Februar 2016

    @noonscoomo: “Das gilt dann ja bitte auch für ein anderes schwarzes Loch. Wie kann es dann sein, dass wir in der Messung sehen, die beiden SLs zu einem einzigen werden?”

    Die Frage versteh ich nicht. Wieso soll ein schwarzes Loch aus nem schwarzen Loch kommen?

  84. #84 Alderamin
    11. Februar 2016

    @Niels

    Danke. Also die Quintessenz ist, dass im Wesentlichen nur der Raum gestaucht bzw. gestreckt wird.

    Da helfen also nette Formeln für die Schwarzschildlösung überhaupt nicht weiter. Man müsste für jede spezielle Gravitationswelle die nichtlinearen Einsteingleichungen in ihrer ganzen Schönheit lösen.

    Wenn im konkreten Fall der Schwarzschild-Lösung aber schon verschiedene Formeln für Längenverkürzung und Zeitdilatation im Schwerefeld gelten (#66), dann zeigt das m.E. bereits, dass sich auf jeden Fall eine messbare Längenveränderung des Interferometers ergeben würde.

    Man kann, wenn ich das richtig verstanden habe, die Gleichungen auch in Abhängigkeit vom Gravitationspotenzial Φ anstelle des Schwarzschildradius formulieren, wäre das dann eine allgemeinere Form? Φ wird, denke ich, auch innerhalb der Gravitationswelle variieren, nicht wahr? Könnte man damit berechnen, wie sich lokal Zeit und Raum verlängern/verkürzen und dies dann über die Länge des Interferometerarms integrieren, um einen exakteren Wert zu erhalten? Nur so zum Mitdenken, nicht, dass ich das vor hätte…. 😉

  85. #85 noonscoomo
    Berlin
    11. Februar 2016

    @ Niels
    Wow, danke für die ausführliche Antwort. Ich denke ich verstehe so ungefähr. Zumindest verstehe ich, dass es komplex ist. Die einfach Antwort scheint zu sein, der eine Arm wird länger, der andere kürzer und die zeitliche Verzerrung kann ich vernachlässigen, weil die Gravitation der Erde schwach ist. Jedenfalls bin ich mir jetzt sicher, dass da intensiv drüber nachgedacht wurde 😉

    @Florian
    Falsch verstanden. Sorry, unklar ausgedrückt.
    Da verschmelzen zwei schwarze Löcher, sie werden also zu einem. Das heisst aber doch, dass die beiden schwarzen Löcher sich annähern, immer schneller umeinander kreisen und dann das Massezentrum des einen hinter dem Schwarzschildradius des anderen verschwindet (und umgekehrt natürlich auch). Ich hab aber gelesen, dass es von uns aus gesehen unendlich lange dauert, bis etwas hinter dem Schwarzschildradius verschwindet. Wie kann es dann sein, dass wir sehen können, wie aus zwei schwarzen Löchern ein neues, schwereres schwarzes Loch entsteht, wenn der Ringdown von uns aus gesehen unendlich lange dauert?

  86. #86 Alderamin
    11. Februar 2016

    @noonscoomo, Niels

    die zeitliche Verzerrung kann ich vernachlässigen, weil die Gravitation der Erde schwach ist

    Ich hätte die zeitliche Verzerrung eher innerhalb der Welle selbst verortet (wo ja auch die räumliche liegt), nicht im Schwerefeld der Erde, welches über das flach liegende Interferometer ziemlich konstant ist. Vielleicht hat uns Niels da ja missverstanden?

  87. […] auch Science Buster, hat das Thema Gravitationswellen und wo man sie finden kann, in seinem Blog astrodicticum-simplex mal […]

  88. #88 adenosine
    11. Februar 2016

    Wie sicher kann man denn sein, dass nicht ein ganz anderer Effekt für den bisher nur die Phantasie fehl eine solche Längenänderung verursacht (auch 5 sigma?).

  89. #89 Niels
    11. Februar 2016

    @walter

    Kann ich mir eine Gravitationswelle als “Erschütterung” der Gravitation vorstellen und nicht als klassische Welle im Sinne einer Wasserwelle?

    Ja, muss man sich sogar so vorstellen. Wobei Gravitation nach Einstein aber eben Raumzeit-Krümmung ist, wodurch das ziemlich unanschaulich wird.
    Wenn sich die Anordnung der felderzeugenden Massen ändert, muss sich ja auch das Feld ändern. Diese Änderung muss anders als bei Newton dem Feld aber irgendwie “mitgeteilt” werden und das geschieht eben durch Gravitationswellen, die sich mit Lichtgeschwindigkeit ausbreiten.

    @Alderamin

    Also die Quintessenz ist, dass im Wesentlichen nur der Raum gestaucht bzw. gestreckt wird.

    Ja.
    Allerdings gilt das so simpel eben nur für den Fall der schwachen Gravitationswellen im schwachen Erdfeld, nicht allgemein.

    Wenn im konkreten Fall der Schwarzschild-Lösung aber schon verschiedene Formeln für Längenverkürzung und Zeitdilatation im Schwerefeld gelten (#66), dann zeigt das m.E. bereits, dass sich auf jeden Fall eine messbare Längenveränderung des Interferometers ergeben würde.

    Verstehe ich nicht?
    Schwarzschild ist doch kugelsymmetrisch.
    Die von dir genannten Formeln gelten bezüglich eines im Unendlichen ruhenden Beobachters. Die von dir in 66 geschriebene Formel für die Zeit besagt damit:
    Für einen solchen Beobachter vergeht die Eigenzeit t₀. Beobachtet er eine Uhr im Abstand R von Nullpunkt, sieht er auf dieser Uhr die Zeit t ablaufen. t ist dabei kleiner als t₀.

    Bei einem Interferometer befindet sich der ganze Aufbau aber doch im selben Abstand R vom Nullpunkt.
    Wo soll also eine Längenveränderung durch Schwarzschild herkommen?

    Oder meinst du, weil die Gravitationswelle R periodisch leicht verändert? Wie gesagt, für schwache Wellen und schwache “Hintergrundfelder” vernachlässigt man das, weil sowieso schon die Möglichkeit der Linearisierung vorausgesetzt wird.

    Man kann, wenn ich das richtig verstanden habe, die Gleichungen auch in Abhängigkeit vom Gravitationspotenzial Φ anstelle des Schwarzschildradius formulieren, wäre das dann eine allgemeinere Form?

    Nein, das ist dann die Newtonsche Näherung der linearen Feldgleichungen, also eigentlich eine Näherung einer Näherung.

    Der Energie-Impuls-Tensor hat die Form
    https://tinyurl.com/jccd3rv

    Wenn alle Geschwindigkeiten klein im Vergleich zur Lichtgeschwindigkeit sind (wovon man im Sonnensystem für Orbitalbahnen ausgehen kann), ist die (Ruhe-)Energie E=mc^2 und damit die Energiedichte, also der T_00 – Anteil, viel größer als alle anderen Einträge. Diese setzt man deswegen als Näherung einfach Null.
    Also krümmt hier nur die Masse die Raumzeit, Impulse, Drücke und Spannungen werden vernachlässigt.

    Die Energiedichte bzw. die Masse kann man dann auch nach Newton in Abhängigkeit vom Gravitationspotential schreiben (Poissongleichung) und das in die Metrik einsetzen, also R durch Φ ersetzen.

    Diese Näherung ist eine Näherung nullter Ordnung und beschreibt skalare Störungen. In dieser Näherung gibt es weder Gravitationswellen noch ein Mitziehen der Raumzeit um rotierende Körper.
    Dafür aber schon Lichtablenkung, Längenkontraktion, Zeitdilation usw.
    Für die Gravity Probe B – Ergebnisse hätte man damit nur die Präzision durch die Raumzeit-Krümmung (6601,8 ± 18,3) vorhersagen können.
    Um die zusätzliche Präzision durch die Rotation der Erde (37,2 ± 7,2) auszurechnen, muss man dann zur nächsten Ordnung de Näherung übergehen.

    In der nächsten Ordnung, also der Näherung erster Ordnung der linearen Feldgleichungen, ist dann auch die Impulsdichte ungleich Null, also die T_01, T_02 und T_03 – Terme. Man hat also Vektor-Störungen.
    (Man beschreibt Impulse ja mit Hilfe von Vektoren.)
    Die Geschwindigkeiten müssen nicht mehr klein im Vergleich zur Lichtgeschwindigkeit sein.
    In dieser Näherung gibt es dann zusätzlich den Effekt des Mitziehens der Raumzeit um rotierende Körper, also sogenannte gravitomagnetische Effekte wie den Lense-Thirring-Effekt.

    Berücksichtigt man alle Einträge im Energie-Impuls-Tensor kommen Gravitationswellen hinzu.
    Ist man im Vakuum, ist natürlich die Energie-Dichte und die Impuls-Dichte Null. Es ist ja keine Materie vorhanden, die Energie und Impuls tragen könnte.
    Dann bleiben nur die Impulsstrom-Einträge ungleich Null, man hat also Tensorstörungen. (Der Impulsstrom wird durch eine 3×3-Matrix beschrieben, das ist eine spezielle Tensor-Form.)
    Wahrscheinlich hast du den Begriff Tensorstörungen im Zusammenhang mit Gravitationswellen schon einmal gehört. Bei der Erklärung der Polarisation der Hintergrundstrahlung bei BICEPs hat das zum Beispiel eine Rolle gespielt.

    (Man spricht von nullter, erster und zweiter Ordnung der linearen Näherung, weil in der T_00- Komponente bei bestimmter Schreibweise c gar nicht auftaucht.
    In den T_01 bis T_03 bzw. T_10 bis T_30 (der Tensor ist symmetrisch, als T_01 = T_10 usw.) Komponenten steht dann v/c und in den restlichen Komponenten schließlich [v^2]/ [c^2].
    Da die Geschwindigkeiten der felderzeugenden Körper durch c bzw c^2 dividiert werden, sieht man, warum man diese Beiträge in der Regel in guter Näherung vernachlässigen kann.)

    @noonscoomo

    Die einfach Antwort scheint zu sein, der eine Arm wird länger, der andere kürzer und die zeitliche Verzerrung kann ich vernachlässigen, weil die Gravitation der Erde schwach ist.

    Richtig.
    Zusätzlich muss die Gravitationswelle ebenfalls schwach sein, aber das habe ich für Gravitationswellen, die die Erde treffen, stillschweigend vorausgesetzt.
    (Wenn im Mondabstand zwei große schwarze Locher kollidieren, haben wir ganz andere Probleme als Gravitationswellen.)

    Das heisst aber doch, dass die beiden schwarzen Löcher sich annähern, immer schneller umeinander kreisen und dann das Massezentrum des einen hinter dem Schwarzschildradius des anderen verschwindet (und umgekehrt natürlich auch).

    Nö, die beiden Ereignishorizonte berühren sich und verschmelzen dann einfach miteinander.
    Was innerhalb des Horizonts passiert, geht uns nix an. 😉

    @Alderamin

    Ich hätte die zeitliche Verzerrung eher innerhalb der Welle selbst verortet (wo ja auch die räumliche liegt), nicht im Schwerefeld der Erde, welches über das flach liegende Interferometer ziemlich konstant ist.

    Das ist die Sache mit der Nichtlinearität.
    Wenn man linearisiert will, müssen beide Summanden klein genug sein. Für die Gravitationswelle im Interferometer habe ich das stillschweigend vorausgesetzt, hätte ich aber hinschreiben müssen.
    In unmittelbarer Nähe des Erzeugungsortes gilt das natürlich auch für die Gravitationswelle nicht.

    Bei einer schwachen Gravitationswelle im starken “Hintergrundfeld” können genau wie bei einer starken Gravitationswelle im schwachen “Hintergrundfeld” durch die Nichtlinearität bei der Wechselwirkung von Welle und Hintergrundfeld auch dann zeitliche Verzerrung auftreten, wenn es allein durch die Gravitationswelle und allein durch das “Hintergrundfeld” keine zeitlichen Verzerrungen geben würde.
    Deswegen sind nichtlineare Differentialgleichung so ne doofe Sache.

    Starke Gravitationswellen in einem schwachen “Hintergrundfeld” kann es aber meiner Meinung nach vermutlich nicht geben, weil alles, das starke Gravitationswellen erzeugen kann, auch ein starkes Feld haben muss. Deswegen habe ich schwache Wellen ohne Nachzudenken und ohne es mitzuteilen als Standard angenommen und nur geschrieben, dass das “Hintergrundfeld” schwach sein muss. Bei schwachem Hintergrunfeld müssen etwaige Wellen schließlich automatisch und zwangsläufig schwach sein.
    (Okay, vermutlich könnte man da auch irgendwie mit Wurmlöchern rumtricksen und dann stimmt es doch nicht absolut immer…)

    @adenosine

    Na ja, wenn man ausgerechnet hat, dass man sich mit 5 sigma sicher ist, dann ist man sich mit 5 sigma sicher?

  90. #90 Niels
    11. Februar 2016

    @walter @Alderamin @noonscoomo
    Und wieder etwas Längeres als #89 in der Moderation.

  91. #92 Krypto
    11. Februar 2016

    Unglaublich:
    Die haben tatsächlich vor dem Regelbetrieb die Verschmelzung von 2 SL mit insgesamt 62 Sonnenmassen aus Richtung GMC in über 1 Mrd. LJ Entfernung detektiert!
    Und das Signal passt perfekt zu den Vorhersagen und Simulationen!
    Wahnsinn!!!

  92. […] Astrodicticum Simplex: Der direkte Nachweis von Gravitationswellen, Was können und wozu braucht man Gravitationswellen? Von Florian […]

  93. #94 PDP10
    11. Februar 2016

    @ambrosia:

    Auch dir sei folgendes ans Herz gelegt:

    Das du etwas nicht verstehst, heisst nicht, dass es falsch oder irrelevant wäre.

    Es bedeutet nur, dass du es nicht verstehst.

  94. #95 noonscoomo
    11. Februar 2016

    @Niels
    Grossartig, dass du dir so viel Zeit nimmst dich mit unseren vermutlich banalen Fragen zu beschäftigen. Danke dafür.

    Das heisst aber doch, dass die beiden schwarzen Löcher sich annähern, immer schneller umeinander kreisen und dann das Massezentrum des einen hinter dem Schwarzschildradius des anderen verschwindet (und umgekehrt natürlich auch).

    Nö, die beiden Ereignishorizonte berühren sich und verschmelzen dann einfach miteinander.
    Was innerhalb des Horizonts passiert, geht uns nix an.

    Hm, irgendwie kauf’ ich das nicht. Nehmen wir mal zwei SL an, die sich einander annähern. Wählen wir nun einen Punkt der auf dem Ereignishorizont des einen SL liegt und nennen ihn A. Dieser Ort hat eine (theoretische) Eigenschaft, an ihm vergeht, von einem entfernten Beobachter aus gesehen, die Zeit unendlich langsam. Nehmen wir nun einen zweiten Ort, der auf dem Ereignishorizont eines zweiten SL liegt und nennen ihn B. Der hat gleiche Eigenschaft wie A. Die beiden Orte A und B seien so gewählt, dass
    sie je den Punkt beschreiben, an dem sich die beiden SLs berühren werden, wenn sie sich hinreichend angenähert haben. Für einen entfernten Beobachter muss es dann doch unendlich lange dauern, bis A und B sich berühren. Oder hab ich da was grundsätzliches falsch verstanden?

  95. #96 adenosine
    12. Februar 2016

    @Niels
    Ich hab das so verstanden, dass sich die 5 Sigma Aussage nur darauf bezieht, dass das Signal kein Rauschen ist, also nicht auf die Ursache des Signals. Die vermutete physikalische Ursache ist sicherlich eine plausible Hypothese, aber gibt es eine Methode die Zuverlässigkeit solcher konstruierter Hypothesen zu berechnen?

  96. #97 Konni Scheller
    Forchheim, Ort des guten Bieres
    12. Februar 2016

    Ich habe eine Frage, von der ich hoffe, dass sie nicht zu blödsinnig klingt.

    Wenn Gravitationswellen eine Stauchung und Streckung des Raumes “an sich” sind – wie kann man das dann überhaupt messen? Wird, bildlich gesprochen, nicht das Lineal, mit dem ich messen will, genauso gedehnt und gestaucht wie der Raum?

    Natürlich muss ich falsch liegen, das ist mir klar. Ich sehe nur nicht, wo. Vielleicht kann jemand helfen.

  97. #98 Alderamin
    12. Februar 2016

    @Niels

    Verstehe ich nicht?
    Schwarzschild ist doch kugelsymmetrisch.
    Die von dir genannten Formeln gelten bezüglich eines im Unendlichen ruhenden Beobachters. Die von dir in 66 geschriebene Formel für die Zeit besagt damit:
    Für einen solchen Beobachter vergeht die Eigenzeit t₀. Beobachtet er eine Uhr im Abstand R von Nullpunkt, sieht er auf dieser Uhr die Zeit t ablaufen. t ist dabei kleiner als t₀.

    Bei einem Interferometer befindet sich der ganze Aufbau aber doch im selben Abstand R vom Nullpunkt.
    Wo soll also eine Längenveränderung durch Schwarzschild herkommen?

    Weiß nicht, war mir ja auch nicht bekannt, bis ich diesen Link fand:
    https://www.solstice.de/cms/upload/wege/band5/wege5-p2-174-187.pdf

    Und da steht unter “6. Verhalten von Maßstäben im Gravitationsfeld”:
    “Ein Maßstab, welcher sich in einem Gravitationsfeld befindet, schrumpft um den Faktor 1-GM/(Rc²)

    Stimmt das also nicht?

    Oder meinst du, weil die Gravitationswelle R periodisch leicht verändert?

    Ich glaube, wir missverstehen uns hier noch einigermaßen, vielleicht fehlt Dir der Zusammenhang. Hier nochmal zusammengefasst:

    Die Ausgangsfrage von noonscoomo war sinngemäß: wenn die Gravitationswelle den Raum verformt, verformt sie dann nicht auch die Zeit, und kann es nicht sein, dass sich diese Verformungen derart gegenseitig aufheben, dass sich keine Phasenverschiebung des Laserlichts ergibt?

    (Gemeint ist: wenn die Länge eines Interfermeterarms durch die Gravitationswelle verkürzt wird, könnte ja der Zeitablauf verlängert werden, so dass effektiv die Lichtgeschwindigkeit nicht mehr die gleiche wäre, was in der ART ja möglich ist – eine Lichtuhr im Schwerefeld tickt langsamer, weil das Licht langsamer zwischen den Spiegeln läuft. Eine Gravitationswelle ist eine periodische Variation des Schwerefelds, könnte also demgemäß auch eine Zeitdilatation verursachen, jedenfalls ist das eine Idee, auf die so ahnungslose Leute wie noonscoomo und ich kommen könnten; wenn die jetzt den reziproken Wert der Längenverkürzung hätte, würde man gar keine Phasenverschiebung messen)

    Die Antwort ist natürlich “nein!”, sonst würde LIGO ja nicht funktionieren, aber gibt es trotzdem irgendeine Variation des Zeitablaufs, wenn eine Gravitationswelle über einen Ort hingwegläuft?

    Und da fand ich halt, dass die o.g. Gleichung für die Längenänderung diejenige für die Zeitdilatation im Schwarzschild-Fall schon einmal exemplarisch (also in einem speziellen Fall, der erst mal gar nichts mit einer Gravitationswelle zu tun hat; hier sollen dann die beiden Formeln aus 66 gelten, für ein festes Schwerefeld einer Masse M im Abstand R mit einer entsprechenden Zeitdilatation und Längenverkürzung) nicht kompensiert, was den Verdacht nahe legt, dass es dann in anderen Situationen (eben bei einer Gravitationswelle, die über einen Punkt hinweg läuft, meinetwegen auf der Erde, aber auch im freien Fall im All) auch nicht zu einer Aufhebung kommt und somit auf jeden Fall eine Phasenverschiebung resultiert.

    Klar, was ich meine?

    Das wäre natürlich alles hinfällig, wenn es eine solche Längenverkürzung im Schwerefeld gar nicht gibt bzw. eine Gravitationswelle gar keine Zeitdilatation mit sich führt.

  98. #99 Alderamin
    12. Februar 2016

    @Niels

    Das wäre natürlich alles hinfällig, wenn es eine solche Längenverkürzung im Schwerefeld gar nicht gibt bzw. eine Gravitationswelle gar keine Zeitdilatation mit sich führt.

    In #78 hast Du ja schon gesagt, dass letzteres zumindest in erster Näherung zu vernachlässigen ist. Ich versuche hier also nur noch, mein Argument zu retten 😉

    Meinetwegen können wir’s auch dabei belassen.

  99. #100 Bullet
    12. Februar 2016

    @noonscoomo:

    Wählen wir nun einen Punkt der auf dem Ereignishorizont des einen SL liegt und nennen ihn A […], an ihm vergeht, von einem entfernten Beobachter aus gesehen, die Zeit unendlich langsam. Nehmen wir nun einen zweiten Ort,[…] der hat gleiche Eigenschaft wie A. […] Für einen entfernten Beobachter muss es dann doch unendlich lange dauern, bis A und B sich berühren. Oder hab ich da was grundsätzliches falsch verstanden?

    Ich denke, ja. Als kleine Analogie: wenn ich mit einem Laser auf eine Fläche leuchte, kann ich einen Ort definieren, an dem der Strahl die Fläche trifft. Das sei seine Eigenschaft. Jetzt ist das ein Zeitverlangsamungslaser, der bewirkt, daß an dem Ort, auf den der Strahl trifft, die Zeit stehenbleibt. Deiner Aussage zufolge erwartest du jetzt, daß ich nicht zwei dieser Laser nehmen könnte, zwei Punkte auf die Fläche malen und die beiden Punkte zu einem zusammenführen könnte, da ja an den Orten der Punkte keine Zeit vergeht.
    Ereignishorizonte sind keine Objekte. Sie sind lediglich Flächen (also Punktmengen), die als einzige Gemeinsamkeit die Eigenschaft haben, gleich weit vom Gravitationszentrum eines SL entfernt zu sein. Ein Ereignishorizont ist eine geometrische Eigenschaft des Raumes, aber kein Ding, mit dem etwas passieren kann oder eben auch nicht. So wie auch Schatten nicht miteinander kollidieren können.
    Hilft das?

  100. #101 Florian Freistetter
    12. Februar 2016

    @Konni: “Vielleicht kann jemand helfen.”

    Das Licht mit dem die Strecke gemessen wird, ist immer gleich schnell. Wenn sich die Distanz ändert, kommt es früher oder später an als erwartet.

  101. #102 Krypto
    12. Februar 2016

    @nonscoomo:

    Für einen entfernten Beobachter muss es dann doch unendlich lange dauern, bis A und B sich berühren. Oder hab ich da was grundsätzliches falsch verstanden?

    Ich würde eher sagen, Du vergleichst Äpfel mit Birnen. 😉
    Da sind 2 SL mit Ereignishorizonten, Ergosphären, verdrillten Raumzeiten und dem ganzen anderen Gedöns.
    Die hängen da nicht einfach im Raum herum und warten, dass ein Beobachter irgendwas auf sie zufallen sieht,
    sondern umkreisen sich mit annähernd Lichtgeschwindigkeit.
    Sie mögen sich vorher Jahrmillionen umkreist haben.
    Aufgrund der extremen Gravitation geht die Verschmelzung selbt jedoch unglaublich schnell vonstatten.
    Und das entstehende, neue SL nimmt unglaublich schnell eine perfekte Kugelform an, rotiert “gemütlich” vor sich hin für ne verdammt lange Zeit.

  102. #103 Niels
    12. Februar 2016

    @Alderamin

    “Ein Maßstab, welcher sich in einem Gravitationsfeld befindet, schrumpft um den Faktor 1-GM/(Rc²)”
    Stimmt das also nicht?

    Come on, das weißt du doch!

    Kurfassung:
    Maßstabe bleiben natürlich gleich, der Raum ist gekrümmt.

    Langfassung:
    In der Schwarzschildmetrik ist der Abstand zwischen den radialen Koordinaten r = 5 und r = 10 nicht einfach 10-5 = 5 wie im flachen, euklidischen Raum. Stattdessen ist er größer als 5, weil der Raum gekrümmt ist.

    Wenn du ein Beobachter bist, der weit weg von der Erde im flachen Raum lebt, kannst du Beobachtungen nahe der Erde auf zwei Arten interpretieren:

    1) Die Erde krümmt die Raumzeit, der Raum ist also gekrümmt.

    2) Der Raum ist immer noch der euklidische Raum.
    A) Benutzt man den Urmeter als Maßstab, muss also die Länge des Urmeters im Gravitationsfeld schrumpfen. Dann ist der Abstand zwischen r = 5 und r = 10 ebenfalls größer als 5.
    B) Benutzt man Licht zur Abstandsmessung, ist die Lichtgeschwindigkeit abhängig vom Gravitationsfeld. Licht breitet sich im Gravitationsfeld mit Geschwindigkeiten kleiner als c aus. Man muss dem Gravitationsfeld also einen Brechungsindex zuordnen.

    “Wege in der Physikdidaktik” verwendet den didaktisch blödsinnigen Standpunkt 2.
    Siehe auch Seite 186 dort:
    Für die Deutung des Resultates gibt es zwei Möglichkeiten:
    Deutung 1: […] Die Struktur des Raumes in der Umgebung der Masse ist somit festgelegt (euklidischer Raum), durch Experimente […] muss Auskunft über das Verhalten der Maßstäbe […] gesucht werden.
    Deutung 2: Jeder Maßstab hat immer die gleiche Länge, gleichgültig Position des Raumes er sich befindet. […] Der die Masse umgebende Raum ist gekrümmt[…]

    Welche Deutung ich für physikalisch richtig halte, ist glaube ich kein Geheimnis.

    Für die Schwarzschildraumzeit kann man die Raumkrümmung sehr einfach angeben, wenn man die Metrik kennt.
    Genau dazu ist die Metrik ja da, sie gibt an, wie in der Raumzeit Entfernungen und Zeitdauern zu berechnen sind.

    Den Abstand zwischen den Radialkoordinaten r^2 und (r+dr)^2 erhält man einfach durch Einsetzen und Subtrahieren in den Metrikanteil der Schwarzschildmetrik ds^2 = 1/([1-2*G*M/(r*c²)].

    Also schlicht dR^2 = { 1/([1-2*G*M/(r*c²)]} * dr^2.
    Ist eigentlich simpel direkt aus der Metrik abzulesen.
    Der radiale Abstand dR zwischen den radialen Schwarzschildkoodinaten A=r und B=r+dr ist also dR, wobei man aus der Formel schließen kann, dass dR größer als dr ist.
    Soviel zu “Eine Ableitung der Gleichung ist mit mathematischen Schulkenntnissen nicht möglich.”

  103. #104 Niels
    12. Februar 2016

    @noonscoomo

    Wählen wir nun einen Punkt der auf dem Ereignishorizont des einen SL liegt und nennen ihn A. Dieser Ort hat eine (theoretische) Eigenschaft, an ihm vergeht, von einem entfernten Beobachter aus gesehen, die Zeit unendlich langsam.

    Das ist so nicht ganz richtig und eine ganz bekannte Verständnisschwierigkeit.
    Das Licht, das direkt am Ereignishorizont abgestrahlt wird, ist für den entfernten Beobachter unendlich rotverschoben. Die Schwarzschildkoordinaten haben als Zeitkoordinate die Eigenzeit eines unendlich weit entfernten Beobachters, sie beschreiben also nicht die tatsächliche Bewegung von Objekten, sondern die Bewegung, wie sie der weit entfernte Beobachter wahrnimmt.

    Dazu hab ich vor einiger Zeit mal etwas Ausführlicheres geschrieben, das verlinke ich einfach mal:
    https://scienceblogs.de/hier-wohnen-drachen/2012/08/19/roger-penroses-zyklisches-universum/#comment-11725

    Siehe auch hier:
    https://math.ucr.edu/home/baez/physics/Relativity/BlackHoles/fall_in.html

    @adenosine

    Na ja, man hat mit 5 sigma ausgeschlossen, dass es ein bekannter Störeffekt ist.
    Natürlich könnte es rein prinzipiell noch eine bisher völlig unbekannte Physik geben, die genau das erwartete Ergebnis liefert, aber eigentlich auf etwas völlig anderes zurückgeht. Besonders wahrscheinlich ist das aber natürlich nicht.

    Man hat aber keine seriöse Möglichkeit, die Wahrscheinlichkeit für den möglichen Einfluss etwas einem bisher völlig Unbekannten sinnvoll abzuschätzen, oder?
    Man könnte versuchen, irgendwie aus der Genauigkeit der vielen, vielen bisherigen Bestätigungen der ART eine Zahl zu konstruieren, da kann ich aber fast genau so gut einfach 10^-50 raten. 😉

    Ist ähnlich wie die Frage, ob das Einschalten des LHC die Erde hätte vernichten können.
    Nach heutigem physikalischen Wissen auf gar keinen Fall, aber 100%ige Sicherheit gibt es in der Physik nicht. Vielleicht errechnet sich mit der Physik des 30. Jahrhunderts doch eine unfassbar winzige Chance, wer weiß?
    Wie könnte man solchen prinzipiell möglichen zukünftigen Erkenntnisfortschritten eine belastbare Wahrscheinlichkeitsangabe zuordnen?

  104. #105 Karl Mistelberger
    12. Februar 2016

    > #96adenosine, 12. Februar 2016
    > Ich hab das so verstanden, dass sich die 5 Sigma Aussage nur darauf bezieht, dass das Signal kein Rauschen ist, also nicht auf die Ursache des Signals. Die vermutete physikalische Ursache ist sicherlich eine plausible Hypothese, aber gibt es eine Methode die Zuverlässigkeit solcher konstruierter Hypothesen zu berechnen?

    Die Allgemeine Relativitätstheorie muss sich der Realität stellen. Diese Bewährungsprobe hat sie bisher mit verblüffender Zuverlässigkeit bestanden:

    The status of experimental tests of general relativity and of theoretical frameworks for analyzing them is reviewed and updated. Einstein’s equivalence principle (EEP) is well supported by experiments such as the Eötvös experiment, tests of local Lorentz invariance and clock experiments. Ongoing tests of EEP and of the inverse square law are searching for new interactions arising from unification or quantum gravity. Tests of general relativity at the post-Newtonian level have reached high precision, including the light deflection, the Shapiro time delay, the perihelion advance of Mercury, the Nordtvedt effect in lunar motion, and frame-dragging. Gravitational wave damping has been detected in an amount that agrees with general relativity to better than half a percent using the Hulse–Taylor binary pulsar, and a growing family of other binary pulsar systems is yielding new tests, especially of strong-field effects. Current and future tests of relativity will center on strong gravity and gravitational waves.

    https://relativity.livingreviews.org/Articles/lrr-2014-4/

    Bleibt hinzuzufügen: Das Ereignis vom 14. September 2015 basierte auf extrem starker Gravitation und Einsteins Theorie beschreibt es perfekt.

    Einsteins Überlegungen gingen von der Äquivalenz von träger und schwerer Masse aus. Seine Neuformulierung der Gravitationstheorie hat nicht nur das damals auftretende Problem der Merkurbahn gelöst sondern beschreibt alle seither gefundenen experimentellen Befunde korrekt.

    It would have been wonderful to watch Einstein’s face had we been able to tell him

  105. #106 Karl Mistelberger
    12. Februar 2016

    > Ihr Kommentar wird moderiert.

  106. #107 noonscoomo
    Berlin
    12. Februar 2016

    @Niels
    Oooh, ach na klar. Der Groschen ist gefallen. Danke. Das war mir in der Tat entgangen. Das heisst also, dass Objekte tatsächlich in endlicher Zeit in das schwarze Loch reinfliegen und dessen Masse erhöhen, nur “sehen” können wir das eben nicht. Aber das heisst dann doch auch, dass ich das Universum nicht unendlich schnell altern sehe, wenn ich selbst in ein SL falle, sondern nur, dass das Licht der Sterne um mich herum dann sehr kurzwellig und verzerrt wird. Richtig?

  107. […] andere astronomische Themen – das Blog Astrodictium Simplex von Florian Freistätter: “Der direkte Nachweis von Gravitationswellen” und “Was können und wozu braucht man […]

  108. #109 Alderamin
    13. Februar 2016

    @Niels

    Come on, das weißt du doch!

    Nicht schimpfen… ich hab’ keine ART in Physik gelernt, ich lerne hier immer noch dazu.

    Die Längenverkürzung gilt also nur radial. Da es sich hier um die kugelsymmetrische Schwarzschildlösung handelt, noch eine Frage: wie sähe es in einem konstanten Feld aus, wo das Gravitationspotenzial überall gleich groß wäre? Wäre da dR auch größer als dr? Oder gilt das nur bei einem Gradienten des Felds?

  109. #110 Niels
    13. Februar 2016

    @noonscoomo

    Aber das heisst dann doch auch, dass ich das Universum nicht unendlich schnell altern sehe, wenn ich selbst in ein SL falle, sondern nur, dass das Licht der Sterne um mich herum dann sehr kurzwellig und verzerrt wird. Richtig?

    Richtig.
    Ein ins schwarze Loch fallender Beobachter sieht nur einen endlichen Abschnitt der Zukunft des Universums, bevor er in die Singularität knallt.
    Dass er die gesamte Zukunft des Universums erlebt und beobachtet ist ein populärer Irrtum.

    @Alderamin

    wie sähe es in einem konstanten Feld aus, wo das Gravitationspotenzial überall gleich groß wäre? Wäre da dR auch größer als dr? Oder gilt das nur bei einem Gradienten des Felds?

    Ja, da wäre dR größer als dr.

    Wobei man eben aufpassen muss, das Gravitationspotential kann man wie oben beschrieben nur in der newtonsche Näherung der linearen Feldgleichungen sinnvoll verwenden, wenn also die Gravitationsfelder sehr schwach sind und wenn man im Energie-Impuls-Tensor alle Komponenten außer der T_00-Komponente vernachlässigen kann, also alles außer der Ruhemasse vernachlässigt.

    Richtiger ist es daher, statt dem Potential von der Krümmung zu sprechen. Es kommt natürlich nur auf die Krümmung genau am betrachteten Raum(zeit)punkt an.
    Im Gravitationspotentialbild spielt der Gradient des Felds deswegen auch keine Rolle.
    Wichtig wäre das dann für Gezeitenkräfte. Beim Fall ins schwarze Loch verursacht die starke Änderung der Krümmung bekanntlich die berühmte Spaghettifizierung.

    Ein konstantes Gravitationsfeld nennt man in der ART lieber einen Raum (keine Raumzeit) mit konstanter Krümmung. Die Krümmung des Raumes ist zu einer festen Zeit t also an jedem Punkte des Raumes gleich.

    Das berühmteste Bespiel dafür sind die Friedmann-Lösungen, also die Metriken, die unser Universum beschreiben.
    Deswegen kann man Universen dann in sphärisch, hyperbolisch und flach unterteilen, ohne konstante Krümmung wäre das nicht nötig.
    Für unser Universum spielt das aber zufällig(?) keine Rolle, da die Krümmung entweder Null oder sehr, sehr nahe Null ist.

  110. #111 Alderamin
    13. Februar 2016

    @Niels

    Danke, macht Sinn.

  111. #112 Karl Mistelberger
    13. Februar 2016
  112. […] Entdeckung? Darum wirds in diesem Artikel gehen. Gleich vorweg möchte ich auch die Lektüre von Florian Freistetters Artikel zum Thema Gravitationswellen ans Herz legen. Sehr […]

  113. #114 klaus
    16. Februar 2016

    Gilt Aktio = Reaktio noch oder wiegen die Gravitationswellen schwerer und bedingen deshalb Aktio # Reaktio ?

    In letzterem Fall könnte ich ein Patent für einen neuartigen Antrieb für “alles” anmelden. Ohne Energieverbrauch. Ohne bewegte Teile. Pure Zugkraft.

  114. […] Tagen haben die Gravitationswellen die Welt der Wissenschaft und auch mein Blog (zum Beispiel hier, hier oder hier) dominiert. Aber natürlich ist die Forschung nicht stehen geblieben und auch […]

  115. #116 Hans Becker
    Linden
    18. Februar 2016

    Zitat:”Eine Gravitationswelle, die sich ja nur mit Lichtgeschwindigkeit bewegt, kommt dort ein klein wenig später an.” Ja, aber doch nur, wenn Anlage 1 und 2 je nach Erddrehung usw. in einer Linie zum Wellenursprung liegen. Wie muss man sich das vorstellen, wie lange “funkt” die Welle denn und was hat sie für eine Frequenz?
    Kann man sie sich wie eine Wechselspannung als Sinus mit einem positiven (dehnenden) und negativen (stauchenden) Teil und “Nulldurchgang” vorstellen?
    Bitte erhellen Sie mich als Laie.

  116. #117 PDP10
    18. Februar 2016

    @Hans Becker:

    Wie muss man sich das vorstellen, wie lange “funkt” die Welle denn und was hat sie für eine Frequenz?

    Hier ist eins der vielen Bilder des gemessenen Signals:

    https://www.faz.net/aktuell/wissen/weltraum/bruce-allen-vom-max-planck-institut-ueber-gravitationswellen-14072285/so-sah-das-vom-ligo-detektor-14076516.html#fotobox_1_4072285

    Das mit dem Sinus ist zwar richtig, aber wie man sieht, sieht man eher einen “Ping” als eine ewig durchlaufende Welle.

    Man sieht auf dem Diagramm die beiden Signale übereinandergelegt. Dabei geht es vor allem darum, ob beide das gleich Signal gemessen haben.
    Haben sie.

    Das Interview zu dem das Bild gehört mit einem der Direktoren des AEI beantwortet übrigens auch ihre Fragen nach Frequenz etc. und ist sehr lesenswert.

    Ja, aber doch nur, wenn Anlage 1 und 2 je nach Erddrehung usw. in einer Linie zum Wellenursprung liegen.

    Nein. Sie können sich die Gravitationswelle – so wie bei jeder Quelle die hinreichend weit entfernt ist – als ebene Welle vorstellen. Würden beide Detektoren in einer Ebene mit dem “Wellenkamm” liegen, hätten sie das Signal halt gleichzeitig gemessen.
    So kam die Welle aber nicht ganz gleichzeitig an. Das wäre auch bei einem schiefen Winkel der Fall – wobei ich jetzt gar nicht weiss, in welchem Winkel der “Wellenkamm” zur Verbindungslinie zwischen den Detektoren lag – ist aber wie gesagt auch nicht wichtig.

    Die Erdrehung spielt dabei übrigens überhaupt keine Rolle. Viel zu langsam gegenüber etwas das sich mit Lichtgeschwindigkeit bewegt.

  117. #118 Hans Becker
    Linden
    18. Februar 2016

    @PDP10:
    vielen Dank für den Link zum Bild, wenn ich die s Scala richtig sehe, haben wir es mit einer Frequenz um die 100 Hz zu tun?
    Mit “auf einer Linie” haben Sie mich evtl. missverstanden: nur angenommen, die Welle käme exakt aus N und die Anlage 1 und 2 lägen in N-S Richtung 3000 km entfernt, hätte ich eine Laufzeitabweichung von 0,01 s, lägen sie in O-W Richtung 3000km entfernt, hätte ich keine.
    Die Hauptanlage arbeitet doch damit, dass der Tunnel 1 anders als der im 90° Winkel angeordnete Tunnel 2 gedehnt / gestaucht wird oder verstehe ich es ganz miss, ich bin ja Laie :-).
    Mit freundlichen Grüßen
    HB

  118. #119 PDP10
    18. Februar 2016

    @Hans Becker:

    Die Hauptanlage arbeitet doch damit, dass der Tunnel 1 anders als der im 90° Winkel angeordnete Tunnel 2 gedehnt / gestaucht wird

    Damit arbeiten beide Anlagen.

    Hätte man drei statt zwei Anlagen, hätte man sogar eine brauchbare Peilung durchführen können.
    So weiss man aber immerhin, dass beide Anlagen das gleiche gemessen haben – was einen groben Messfehler ausschließt.

    Wie gesagt, es lohnt sich, das Interview zum Bild bei der FAZ Online zu lesen und auch die Diskussion hier und im Zugehörigen Artikel:

    https://scienceblogs.de/astrodicticum-simplex/2016/02/11/was-koennen-und-wozu-braucht-man-gravitationswellen/

    Da wird eine Menge von Leuten erklärt, die davon viel mehr verstehen als ich.

  119. #120 Alderamin
    18. Februar 2016

    @PDP10

    Okay das ist ein Argument. Aber für den Fall dass mein Kopf schwerer ist als meine Füße würde ich eher kürzer als länger oder?

    Die Detektoren sind 3000 km voneinander entfernt, das macht eine Lichtlaufzeit von höchstens 10 ms. Gemessen wurde ein Laufzeitunterschied von 7 ms, demnach müsste der Winkel etwa asin (7/10) = 44° betragen haben.

    Wenn es in der Presse hieß, die Wellen wären im “genau richtigen Zeitabstand” empfangen worden, dann hieß das, dass sie höchstens mit 10 ms Unterschied hätten registriert werden dürfen, sonst hätte es sich beispielsweise um ein Erdbeben oder eine lokale Störung handeln können.

  120. #121 PDP10
    18. Februar 2016

    @Alderamin:

    Du bist entweder im falschen Thread oder hast den falschen angesprochen.

    Öhm … eigentlich wahrscheinlich beides :-).

  121. #122 Alderamin
    18. Februar 2016

    @PDP10

    Huch, da hat die Tastenkombination ctrl-c wohl versagt und ctrl-v den Text vom vorherigen Kopieren eingesetzt. Mein Post bezog sich auf Deinen Satz:

    So kam die Welle aber nicht ganz gleichzeitig an. Das wäre auch bei einem schiefen Winkel der Fall – wobei ich jetzt gar nicht weiss, in welchem Winkel der “Wellenkamm” zur Verbindungslinie zwischen den Detektoren lag

  122. #123 PDP10
    18. Februar 2016

    @Alderamin:

    Ah, Ok. Jetzt ist der Zusammenhang klar :-).

    (Kopf, Füsse, kürzer, länger … hä?!? 😉 )

  123. #124 Alderamin
    19. Februar 2016

    @PDP10

    Das kam von hier.

  124. #125 Dr.Goltz
    21. Februar 2016

    wenigstens etwas an physikalischen Fragen interessiert seiend, ich bin Dipl.-Ing.der Elektrotechnik, zum Dr.phil. promoviert habend, habe ich im vorletztem Archivband meiner Homepage, erreichbar wie folgt: http://www.evolution-of-nature-in-german.com/ versucht das Phänomen der Gravitation mittels atomphysikalischer Detailbetrachtung zu erklären, der Ansicht folgend, dass im tiefsten Grund der Dinge bloss Masse da ist, einerseits,andererseits Antimasse.Diese etwas unscharfe Bemerkung soll eins signalisieren: Mathematik ist sowas wie ein Denkkleid, beschreiben wir mit etwas aus der Mathematik die Welt, gehen wir zum tiefsten Grund der Dinge überhaupt,bleibt nur die verbale Beschreibung.

  125. #126 Dr.Goltz
    21. Februar 2016
  126. #127 Krypto
    21. Februar 2016

    @Goltz:
    Nicht schon wieder ein Inschinöör ;(
    Und Blogs sind auch keine Werbefläche für Privattheorien.

  127. #128 ambrosia
    Kirchheim
    21. Februar 2016

    Jetzt wartet doch erst ab -statt in irgendwelche Hysterien zu verfallen … das ganze muss mehrmals gegengecheckt werden – wie gesagt Blamagen gab es bereits genug.

  128. #129 Florian Freistetter
    21. Februar 2016

    @ambrosia: “das ganze muss mehrmals gegengecheckt werden”

    Du meinst, so mit unterschiedlichen Observatorien, ausführlicher statistischer Datenanalyse, usw? Also so, wie es in diesem Fall geschehen ist, bevor man die Ergebnisse veröffentlicht hat?

  129. #130 ambrosia
    Kirchheim
    21. Februar 2016

    Ich denke, man wird diesmal seeehr gründlich vorgehen wollen, um wirklich alle Eventualitäten auszuschliessen.
    Zu ärgerlich, wenn wieder ein Lapsus passiert wäre.
    Und “einfacher” Faktencheck ist nicht so ohne, wenn der Erwartungsdruck einen selbst zensiert und man immer nur den Zusammenhang sieht, den man sehen will. Auch den seriösesten Wissenschaftlern ist das schon unterlaufen – es menschelt eben, wir sind keine Maschinen.

  130. #131 ambrosia
    21. Februar 2016

    Ach ja, und noch was – Arroganz steht Dir nicht (immer ein Zeichen von Schwäche) – ich bin seit 30 Jahren Physikerin und auch in meinem Beruf tätig – ich denke solche herablasende Spielchen können wir also lassen.

  131. #132 Dietmar
    21. Februar 2016

    Welche Arroganz?

  132. #133 Hans Becker
    Linden
    21. Februar 2016

    Wenn ich mir die Einträge seit Krypto “Nicht schon wieder ein Inschinöör” ansehe, frage ich mich, wie dieses Niveau zum Thema passen soll. Schade eigentlich.

  133. #134 Florian Freistetter
    21. Februar 2016

    @ambrosia: “Ach ja, und noch was – Arroganz steht Dir nicht (immer ein Zeichen von Schwäche) – ich bin seit 30 Jahren Physikerin und auch in meinem Beruf tätig – ich denke solche herablasende Spielchen können wir also lassen.”

    Wo genau wäre ich jetzt “arrogant” gewesen? Vielleicht würde es helfen, wenn du genauer spezifiziert, inwiefern diese Beobachtung deiner Meinung nach unbestätigt ist? Ich jedenfalls kann das nicht erkennen.

  134. #135 Krypto
    21. Februar 2016

    @ambrosia:
    Ein wenig zu dick aufgetragen, würde ich sagen.
    Du bist nie und nimmer Physiker(in), sonst würdest Du nicht so uninformiert daherseiern.

  135. #136 Krypto
    21. Februar 2016

    @Hans:
    Du meintest sicherlich nicht “seit Krypto”, sondern, seit “Dr. Goltz”…

  136. #137 ambrosia
    21. Februar 2016

    http://www.qualitygate-ccr.de: Für das kleine Männer-ego von krypto.
    Arroganz: eine Anrede … Du meinst wohl … ist schon reichlich kindlich.
    “Du meinst, so mit unterschiedlichen Observatorien, ausführlicher statistischer Datenanalyse, usw? Also so, wie es in diesem Fall geschehen ist, bevor man die Ergebnisse veröffentlicht hat?”
    Gehts noch eingbildeter ?

  137. #138 Florian Freistetter
    21. Februar 2016

    @Krypto: “Du bist nie und nimmer Physiker(in), sonst würdest Du nicht so uninformiert daherseiern.”

    Wie üblich: Nicht streiten, vor allem nicht über Dinge, die man nicht wissen kann. Sondern zur Sache diskutieren. Anscheinend haben ich und ambrosia unterschiedliche Auffassungen, was “überprüft” angeht. Das wollte ich klären. Da müssen wir nicht in die – üblichen und immer wieder nervigen – Nebenschauplätze abdriften.

  138. #139 PDP10
    21. Februar 2016

    Gehts noch eingbildeter ?

    Da nimmt sich @ambrosia grad aber nix im Tonfall im Vergleich zu sonstwem …

    @ambrosia:

    Sibylle Anderl hat zu der Art wie die Daten ausgewertet, begutachtet, qualitätskontrolliert usw. wurden einen sehr lesenswerten Blogbeitrag bei der FAZ geschrieben, der hier schon an anderer Stelle diskutiert wurde:

    https://blogs.faz.net/planckton/2016/02/17/gravitationswellen-und-das-entdeckerdilemma-1541/

  139. #140 Dietmar
    21. Februar 2016

    @ambrosia: Du hast jetzt schlecht belegt Arroganz vorgeworfen und sexistisch „kleines Männer-Ego”. Das bist Du jetzt ja nun losgeworden. Schön.

    Was ist jetzt Deine Kritik an dem Forschungsergebnis?

  140. #141 Dr. Goltz
    22. Februar 2016

    Genaugenommen handelt es sich um ein Wagnis, einen weiteren Kommentar einzusetzen, will mir doch nicht so richtig einleuchten, dass es Gravitationswellen überhaupt gibt, eher davon ausgehend, postulativ gesehen, dass das gemessene Signal mit etwas anderem zu tun hat, wie behauptet wurde.Ich gehe davon aus, heuristisch gesehen, dass besagtes Signal nicht aus dem Weltraum kommt, für möglich haltend, dass Fragen der Messtechnik ausreichend sein könnten, das Doppelmuster besagten Signals zu erklären, etwa so: wir kennen nicht alle Parameter, die eine Rolle zu spielen wissen, wir kennen nicht alle Feinstpartikel, somit für möglich haltend, dass besagtes Signal einer “Laune der Natur” entspricht, dann weiter annehmend, dass es sich um einen einmaligen Vorgang handelt, einen Vorgang, der sich nicht zu wiederholen weiss.Sollte ich recht haben, gut, sollte ich ncht recht habend, auch gut.Die Zukunft entscheidet.

  141. #142 Krypto
    22. Februar 2016

    @Goltz:
    Solange Du uns nicht mit Werbung nervst, diskutieren wir gerne mit Dir über das Blogthema.
    @Florian: ambrosia hat nun auch erfolgreich ihre Werbung platziert.

  142. #143 Dr. Goltz
    22. Februar 2016

    Ich persönlich bin gespannt, ob eine erneute Messung von Gravitationswellen vorkommen wird. Einstein gab in einem Artikel an, dass Gravitationswellen wiederum Gravitationswellen produzieren sollten.Was das neulich gemessene Signal anbelangt, könnte es sich bei der Erstmessung um vagabundierende Gravitonen gehandelt habend, bei der Zweitmessung, kurz danach, um Streuungen besagter vagabundierender Gravitonen, Einstein´s Gedanken aufgreifend.

  143. #144 ambrosia
    Kirchheim
    22. Februar 2016

    Lerne erst einmal lesen … es geht nicht um Kritik der Ergebnisse, sondern darum hier keine Hysterie zu verbreiten, sondern abzuwarten und die Leute ohne Erwartungsdruck ihre Arbeit machen zu lassen.
    Was ich hier so lese an Kommentaren zeigt mir, dass es sich bei einigen wohl um reichlich pubertäre Personen handeln muss. Wenn deren Weltbild (Frauen sind blöd – nicht stimmt, und ich bin ja nun mal seit 30 Jahren Physikerin) werden sie unruhig, und der Tonfall, den ich hier so lesen muss, bestätigt das nur. Erinnere Dich, ich wurde sehr polemisch angegriffen – und habe entsprechend geantwortet. Was war noch mal DEINE Ausbildung?

  144. #145 Dietmar
    22. Februar 2016

    @Dr. Goltz: Was ist Dein Fachgebiet?

    Genaugenommen handelt es sich um ein Wagnis, einen weiteren Kommentar einzusetzen, will mir doch nicht so richtig einleuchten, dass es Gravitationswellen überhaupt gibt

    Abgesehen davon, dass unnötig verschachtelte Sätze kein guter Stil sind, insbesondere weil sie Intellektualität vorgaukeln sollen und deshalb hier ein Punkt angezeigt gewesen wäre: Du hast außer kompliziert ausgedrücktem „Ich glaube das nicht!” nichts gesagt. Warum machst Du so etwas?

  145. #146 ambrosia
    Kirchheim
    22. Februar 2016

    Krypto, Du bist wahrscheinlich irgendwie gestört – mir meiner reichlich alten Webseite, deren Gestaltung eher abschreckend ist, muss ich wirklich keine Werbung betreiben. Meine Aufträge bekomme ich über technische und industrielle Gremien wie der DVS und der IMAPS und ähnlichen Verbänden. Eine Mitgliedschaft hier ist wesentlich nützlicher als ein Webauftritt, den braucht man nur für den Businessplan, um irgendeine Werbeaktivität nachzuweisen, weil das nun mal Vorschrift ist.
    Ach ja, und wenn Du deine Aufmerksamkeit auf die beiden Organisationen richten willst, dann kannst Du mir ja hier auch gleich wieder Werbung vorwerfen – was für ein Kindergarten ist das hier eigentlich ?

  146. #147 Dietmar
    22. Februar 2016

    @ambrosia: Ich vermute, dass Du mit Deinem letzten Kommentar mich ansprichst. Jetzt kommt bei Dir zu den sexistischen Angriffen noch Beleidigungen in Form von Leseschwäche und Reifeverzögerung. Ich kann Dir gegenüber aber nirgends gleiches sehen.

    Du willst also sagen, man soll erst mal warten und die Ergebnisse verifizieren. Das hat man aber schon getan und jetzt gehen andere an die gleiche Fragestellung heran. Als Physikerin, das hast Du in meinen Augen bisher allerdings nur wenig überzeugend behauptet, solltest Du wissen, wie das funktioniert in der Wissenschaft.

    Ich bin Musiker und Lehrer und erfinde keine Qualifikationen, die ich nicht habe, um meinen Worten mehr Gewicht zu verleihen. Ich benutze auch keine sexistischen Pauschalvorwürfe.

  147. #148 Florian Freistetter
    22. Februar 2016

    @ambrosia:“Du bist wahrscheinlich irgendwie gestört “

    Auch für dich gilt: Lass die persönlichen Beleidigungen und diskutiere zur Sache. Oder gar nicht. Du hast immer noch auf die Frage geantwortet, was genau deiner Meinung nach an der Detektion der Gravitationswellen noch “ungeprüft” ist.

  148. #149 Dietmar
    22. Februar 2016

    @ambrosia: Du hast Dein „Wartet mal ab!” ja nun gesagt. Wurde wohl von jedem verstanden. Weil außerdem nur Pöbelei von Dir kommt, kann ich Dich ja ignorieren ohne etwas zu verpassen.

    Aber wenn Du Dich nicht zusammennimmst, wirst Du hier wohl zeitnah rausfliegen.

  149. #150 ambrosia
    22. Februar 2016

    Ihr solltet euch in scientologyblogs umbenennen – mit den Beleidigungen und Provokationen fangt Ihr an, wenn sich Leute wehren, werdet Ihr bösartig.
    Sinnvoll diskutieren kann mit mit solchen Menschen nicht.
    Bleibt also unter Euch – einer Art Sekte, wie mir scheint.

  150. #151 Dietmar
    22. Februar 2016

    @ambrosia: Da es hier gerade um Qualifikation geht:

    mir meiner reichlich alten Webseite, deren Gestaltung eher abschreckend ist, muss ich wirklich keine Werbung betreiben.

    Im Impressum finde ich nur einen weiblichen Namen: Den der Webdesignerin.

  151. #152 ambrosia
    22. Februar 2016

    Schlauberger – manchmal macht man das halt selbst.
    Schau mal unter Kontakte nach, vielleicht erhellt sich was.
    Aber wie gesagt – mein letzter post auf Eurer merkwürdigen Seite.

  152. #153 Noonscoomo
    Berlun
    22. Februar 2016

    @Goltz
    Es ist vernünftig, erst mal davon auszugehen, dass ein Fehler vorliegt und auch zu berücksichtigen, dass es eine Laune der Natur sein kann. Das wurde in diesem Fall getan. Es gibt eine Wahrscheinlichkeit von weniger als 1/3500000 dass es sich um eine Laune handelt. Übrigens wurden mehrere Signale gemessen, bei den anderen kann man aber eine Laune nicht mit dem erforderlichen 5 Sigma ausschliessen. Auch der ausserordentlich kurze Zeitabstand der Messung an beiden Standorten ist ein guter Hinweis. Andere Arten von Erschütterung reisen nicht so schnell. Nicht mal annähernd.
    Aber, was ich auch nicht genau überblicke, wie schliesst man Betrug aus? Schliesslich gehts hier auch um viel Geld.

  153. #154 Dietmar
    22. Februar 2016

    Ich habe eben mit @ambrosia telefoniert: Sie ist eine sehr nette und engagierte Physikerin, die, das ist nicht abwertend gemeint, ihr Herz auf der Zunge trägt. Ich habe bei dem Gespräch gelernt, dass auf “unserer” Seite offenbar immer noch zu schnell scharf geschossen wird.

    Der Blogchef hat also Recht mit seiner ständigen Forderung, sich zurückzuhalten!

    @Florian Freistetter: @ambrosia wollte zur allgemeinen Vorsicht mahnen und hat keinen konkreten Zweifel, wenn ich sie richtig verstanden habe. Deine Antwort kam bei ihr herablassend an. Ich bin mir sicher, so war der nicht gemeint und das ist auch nicht Deine Haltung.

    Ich denke, @ambrosia wäre eine spannende Kommentatorin hier.

  154. #155 Dietmar
    22. Februar 2016

    Beim nochmaligen Lesen meiner Kommentare kann ich nach dem sehr freundlichen Gespräch sogar verstehen, dass sich @ambrosia auch von mir angegriffen fühlte. Tut mir leid und ich entschuldige mich hier dafür ausdrücklich!

    Eigentlich bin ich viel netter *hüstel* …

  155. #156 Adent
    22. Februar 2016

    @Dietmar/Ambrosia
    Das ist das Problem bei schriftlicher Kommunikation (eigentlich bei jeglicher Kommunikation), bei mündlicher oder gar bei mündlich-optischer Kommunikation kann man meist erkennen wie der andere es gemeint hat, bei schriftlicher gerät es oft in den falschen Kanal. Das beste Beispiel ist immer die Ironie in schriftlicher Form, dahingehend war ja wohl auch Florians Antwort auf Ambrosia (ich habe es als leicht genervte Ironie interpretiert, Ambrosia als eingebildete patzige Antwort).

  156. #157 Dietmar
    22. Februar 2016

    @Adent: Deshalb habe ich zum Hörer gegriffen. Als sie mich als Schlauberger bezeichnete, dachte ich erst “Beleidigung!”, dann, dass es ihr, wenn sie Physikerin ist, das ja auch so vorkommen muss, und letztlich, dass das mir Freunde in Gesprächen an den Kopf werfen könnten und wir darüber höchstens lachen würden. Ich habe nach meiner Entschuldigung hier nochmal angerufen und mich persönlich entschuldigt. Sie nahm das zum Anlass mir unter anderem die Wechselwirkungen von Raumzeit und Gravitation sehr anschaulich und ausführlich ungefragt zu erklären. Sie hat sich richtig Zeit für einen Wildfremden genommen, von dem sie den Eindruck hatte, er wolle sie antrollen!

    Eine sehr sympathische Frau und ich schäme mich tatsächlich für mein Auftreten ihr gegenüber.

  157. #158 Captain E.
    22. Februar 2016

    Dietmar, ich habe gerade noch einmal die letzten Kommentare durchgelesen, kann aber keinen Grund für dich persönlich erkennen, dich zu schämen. Krypto ist ambrosia etwas heftiger (sprich: zu heftig) angegangen, und Florian und ambrosia haben völlig aneinander vorbei geredet. Ihre Gegenreaktion war allerdings meiner Meinung nach auch überzogen, zumal hier wirklich niemand etwas frauenfeindliches gesagt hatte. Die alte Redensart gilt halt noch immer: “Wie man in den Wald ruft, so schallt es zurück.”

    Nachdem nun die Frage nach der Qualifikation von ambrosia geklärt ist, würde ich allerdings gerne von ihr noch hören, ob sie irgendwelche Bedenken über die normale Vorsicht hinaus hegt. Wie Florian schon erwähnt hatte, hat man verschiedene Messergebnisse erhalten und diese gründlich überprüft. Wenn man nicht gerade “Mist gemessen” oder sich knallhart verrechnet hat, scheinen die Resultate belastbar zu sein. Wo hast du also deine Bedenken, ambrosia, dass du meinst, hier vor “Hysterie” warnen zu müssen? Anzeichen von Hysterie habe ich für meinen Teil höchstens bei Kommentatoren wahrgenommen, für die Albert Einstein offenkundig ein Betrüger und Scharlatan gewesen ist.

  158. #159 Florian Freistetter
    22. Februar 2016

    @ambrosia: “Bleibt also unter Euch – einer Art Sekte, wie mir scheint.”

    Es ist schade, dass du meine Frage nicht beantworten möchtest. Du meintest, die Entdeckung der Gravitationswellen wäre noch nicht genau genug überprüft. Ich habe gefragt, was genau du mit so einer Prüfung meinst – denn DASS eine Prüfung stattgefunden hat, hab ich in meiner ersten Antwort ja gesagt. Wenn du darauf antworten würdest, hätten wir eine sinnvolle Diskussion führen können.

    @alle: Es wäre nett, wenn ihr nicht jedesmal sofort davon ausgehen würdet, jemand wäre ein Troll/Spinner/etc. Ich weiß, euch macht es Spaß, solche sinnlosen Streitereien zu führen. Aber mir machts keinen Spaß, ein Blog zu führen, in dem sich jeder sofort bei jeder Kleinigkeit so sehr provoziert fühlt. Lasst euch von nicht immer von den Trollen an der Nase herumführen. Die kann man auch ignorieren.

  159. #160 Florian Freistetter
    22. Februar 2016

    @Dietmar: “Deine Antwort kam bei ihr herablassend an. Ich bin mir sicher, so war der nicht gemeint und das ist auch nicht Deine Haltung.”

    Erst jetzt gelesen. Ja, war so nicht gemeint.

    “Der Blogchef hat also Recht mit seiner ständigen Forderung, sich zurückzuhalten!”

    Was aber erfahrungsgemäß leider kaum jemanden zu interessieren scheint…

  160. #161 Dietmar
    22. Februar 2016

    @Florian Freistetter: Ohne ausufernd meta-diskutieren-zu-wollen:

    Was aber erfahrungsgemäß leider kaum jemanden zu interessieren scheint…

    Das scheint vielleicht nur so. Hättest Du mich eingebremst, hätte ich das wirklich nicht verstanden oder eingesehen. Aber @ambrosia schien mir wirklich betroffen zu sein und hat dann dementsprechend gemeint berechtigt tief in den Köcher mit Giftpfeilen greifen zu müssen.

    Heikel das Ganze. Vor allem wenn es nach meinem jetzigen Eindruck vernünftige Leute trifft.

    @CaptainE.: Danke, aber ich muss mit mir doch nochmal ein ernstes Wort reden…

  161. #162 ambrosia
    22. Februar 2016

    Es ging nicht um die Überprüfung der Ergebnisse an sich, sondern die Selbstüberprüfung. Bei einem Effekt, der auf einem selten auftretenden Ereignis beruht, neigt man schon dazu, sich zu fragen: … too good to be true, warum hatten wir soviel Glück – hatten wir soviel Glück …? etc. Vor allem, wenn solche Entdeckungen gleich zum Welt-(Medien)ereignis werden, ist das durchaus schwierig. Insbesondere, wenn die Erwartung so gross ist, und vorher falsch positive Resultate auch bereits um den Globus gingen.
    Das meine ich mit seehr vorsichtig.

  162. #163 Alderamin
    22. Februar 2016

    @ambrosia

    Weißt, dass das LIGO-System den Forschern gelegentlich schon einmal Fake Events einspielt, um sie zu testen? Das letztemal hatten sie schon eine Veröffentlichung für die Einreichung fertig, da wurden sie erst informiert, das macht sie nun vorsichtig. Sie haben auch schon seit 2002 Erfahrungen mit dem Gerät sammeln können.

    Um nicht auf andere Erschütterungen hereinzufallen, gibt es zig Sensoren um die Geräte herum, z.B. Seismographen. Erschütterungen oberhalb einer bestimmten Fequenz sind durch die vierfach gestaffelte Aufhängung ausgefiltert, darunter wirkt eine aktive Kompensation.

    Hier gibt’s das Transskript eines Interviews mit einem der Beteiligten, der erklärt, wie gründlich das Event vor der Veröffentlichung analysiert wurde.

  163. #164 Captain E.
    22. Februar 2016

    @Ambrosia:

    Da du vom Fach bist, hast du aber doch wohl mitbekommen, mit welchem recht gemeinen Trick bei LIGO gearbeitet wird, oder? Den Wissenschaftlern, die direkt mit den Daten arbeiten, werden ab und zu schon einmal Testresultate eingespielt, deren Signifikanz aber zu niedrig für einen Treffer ist. Wer darauf hereinfällt, ist dann tatsächlich blamiert. Von daher werden die Leute, die entscheiden müssen, ob sie mit Resultaten an die Öffentlichkeit gehen, schon aus reinem (wissenschaftlichen) Selbsterhaltungstrieb extrem vorsichtig vorgehen.

  164. #165 Florian Freistetter
    22. Februar 2016

    @ambrosia: Dich dürfte vielleicht auch dieser Artikel hier interessieren. Da wird die Sache mit der Entdeckung und all den verbundenen Problemen schön beschrieben: https://blogs.faz.net/planckton/2016/02/17/gravitationswellen-und-das-entdeckerdilemma-1541/

  165. #166 ambrosia
    22. Februar 2016

    Ja, das ist genau das Problem – interessanterweise mit einer Methode der Psychologie angegangen – dem Blindversuch – Doppelblind geht ja nicht, weil sonst der Fake-Initiator auch nicht wüsste, dass ein Fake -Impuls abgegeben wurde.
    Der 4-Stunden Fehler – das ist die Nemesis per se: Ein so trivialer “Auslasser”, dass er einem gar nicht auffällt. Das ist, was man auch am meisten fürchtet – der “Überlichtgeschwindigkeits-Wackelkontakt” beim CERN fällt auch da rein.

  166. #167 Hans Becker
    Linden
    22. Februar 2016

    Die Diskussionen von Spezialisten über hochspezielle Fragestellungen sind das eine. Mir (als Laie) würde es sehr weiterhelfen, wenn einer der Experten den Messaufbau genau beschreiben könnte (evtl. in einem gesonderten Artikel), insbesondere, wie man sämtliche Störgroßen, wie seismische Erschütterungen, thermisch bedingte Ausdehnung usw. usf. herausgerechnet hat, die allesamt tausendfach bzw. noch erheblich viel größer als die Messgröße sind. Ich würde die Frage hier nicht stellen, wenn so etwas an anderem Ort zusammenhängend und allgemein verständlich beschrieben wäre, ich finde aber nichts Geeignetes. Was ich bisher zu wissen meine, ist, dass sich die Welle vom Ursprung her kugelförmig ausbreitet, dass sie jede Materie „unbeeindruckt“ passiert und dass ihre Wirkrichtung rechtwinklig zur Ausbreitungsrichtung steht. (?) Wer macht sich die Mühe, dies -vielleicht mit der einen oder anderen Skizze- zu beschreiben?

  167. #168 noonscoomo
    Berlin
    22. Februar 2016

    Mich interessiert nach wie vor die Frage, wie kann man das Potenzial für Betrug minimieren.
    Wenn es möglich ist, fake Daten einzuspielen, welche Möglichkeit haben die Forscher, herauszufinden, ob es welche waren. Offenbar gibt es ja ein Interface für das Einspielen von fake Daten, also kann man das Interface auch hacken. Andere Datennetze und Computer sollen auch schon mal gehacked worden sein, hab ich gehört
    😉
    Und es kann natürlich noch sein, dass authorisierte Personen ein eigenes Interesse hatten, dass die Entdeckung gemacht wird, da dann sicher viel Geld locker gemacht wird um neue Anlagen zu bauen, und mal ein paar Daten eingespielt und dann die Spuren verwischt haben. Evtl. haben sie auch Geld bekommen, um das zu tun. Ich hab gehört, sowas soll auch schon mal in anderen Bereichen passiert sein. Wir wird in diesem Fall konkret der Missbrauch des fake Interfaces durch dritte oder durch Insider verhindert?

  168. #169 ambrosia
    22. Februar 2016

    Naja lasst mal die Kirche im Dorf … Gut das Geld ist oft knapp in der Forschung (leider) aber so verzeifelt wird wohl keiner sein … die Reputation wäre auf ewig dahin.
    Bösartige Hacker haben heutzutage auch eher ihre Geldbörse im Sinn, als Grundlagen-Forschung zu sabotieren.

  169. #170 Alderamin
    22. Februar 2016

    @ambrosia

    Das ist, was man auch am meisten fürchtet – der “Überlichtgeschwindigkeits-Wackelkontakt” beim CERN fällt auch da rein.

    Soviel ich mich erinnern kann, war das damals allerdings keine Meldung “Wir haben die Überlichtgeschwindigkeit entdeckt!” sondern eher “wir messen hier immer Überlichtgeschwindigkeit und finden keinen Fehler, wer kann helfen?”

    Auch Florian hatte sich dereinst sehr vorsichtig darüber geäußert und ich hab’s in den Kommentaren auch nicht wahr haben wollen. Ist hier bei LIGO schon deutlich definitiver.

    Eher warnen sollte BICEP2 mit den vorschnell gemeldeten Polarisationsmustern. Da kam aber bald schon Widerspruch, und dieses Experiment hatten die LIGO-Forscher auch im Hinterkopf, wie ich irgendwo las. Sie wollten nicht in diese Falle tappen.

  170. #171 Florian Freistetter
    22. Februar 2016

    @ambrosia: “Der 4-Stunden Fehler – das ist die Nemesis per se: Ein so trivialer “Auslasser”, dass er einem gar nicht auffällt.”

    Na ja, bei Weber damals wars ja eher weniger ein klassischer Fehler sondern schon mehr übersteigertes Wunschdenken hart am Betrug vorbei geschrammt. Dass Webers Daten nicht mehr ernst zu nehmen sind, hat sich ja bald nach den ersten Überprüfungen gezeigt. Denn natürlich wird immer überprüft…

  171. #172 Rex-Lii
    22. Februar 2016

    Ich finde die Entdeckung von Gravitationswellen sehr interessant und spannend, mal schauen was sich daraus entwickelt. Ich lebe noch ca 60 Jahre +-, da wird sich sicher viel machen!

    Und wieder zeigt sich, die Realität ist so viel spannender als Science Fiction! Denn das hier ist real. Ausdenken kann ich mir vieles, aber die Realität hat einfach einen ganz einzigartigen Scharm.
    Wenn wir irgendwann andere Planten innerhalb oder ausserhalb unseres Sonnensystems erforschen wird das sehr spannend, denn es wird real sein!

  172. #173 Florian Freistetter
    22. Februar 2016

    @Hans Becker “insbesondere, wie man sämtliche Störgroßen, wie seismische Erschütterungen, thermisch bedingte Ausdehnung usw. usf. herausgerechnet hat, die allesamt tausendfach bzw. noch erheblich viel größer als die Messgröße sind.”

    In der aktuellen Folge von “Probably Science” wird ua über dieses Thema gesprochen: https://www.probablyscience.com/probablyscience/2016/2/19/gravitation-wave-ligo-spectacular

  173. #174 Krypto
    22. Februar 2016

    @Hans Becker:
    Schau mal hier:
    https://scienceblogs.de/hier-wohnen-drachen/2016/02/21/noch-einmal-die-raumverzerrung/
    Nebenan hat das Martin nochmal richtig aufgedröselt, was da prinzipiell wie gemessen wird.
    Mehr Infos zum aktuellen Aufbau von Ligo gibt´s auf deren HP, falls Englisch kein Problem für Dich ist:
    https://www.advancedligo.mit.edu/

    @ambrosia:
    Na dann willkommen in den sb-Kommentaren 😉
    Wenn Du Dein Temperament künftig ein wenig zügelst, reduziert das die Trollalarmschwelle erheblich.

  174. #175 Hans Becker
    Linden
    23. Februar 2016

    @Krypto & Freistetter:
    Ihr Verweis auf die genannten Links ist ja schön und gut (ich habe mir das alles durchgelesen mit den Kugeln in 100 cm Abstand und verstehe das auch), das kocht aber doch alles in der gleichen Suppe einer hochspezifischen Herleitung aus der “ART” und gibt mir keinerlei Aufschluss, wie die Jungs bei LIGO und sonstwo die millionenfach größeren Störgrößen aus der Messgröße herausgerechnet haben wollen.
    Interessiert Sie das denn überhaupt nicht oder ist die Fragestellung so kompliziert oder trivial, dass ein Physiker die einem Techniker nicht mehr beantworten kann?
    Ich zweifele ja nicht an der Sache, ich will sie technisch nur verstehen!
    Beste Grüße

  175. #176 Dr. Goltz
    23. Februar 2016

    Nimmt man an, dass es Gravitationswellen gibt, macht man vermutlich einen Fehler, darauf hinweisend, dass selbst Einstein in dieser Richtung schon Zweifel gehegt hat.Die Gleichungen der allgemeinen Relativitätstheorie stellen nichts anderes dar, als zu beschreiben zu wissen, keinesfalls das kleine, die Welt der subatomaren Teilchen, vielmehr das gröbere , etwa die Merkurlaufbahn..Einen Zugang zur Welt der Quantenphysik schafft die ART nicht.Solange eine komplett zu nennende Theorie nicht vorhanden ist, müsste man sich in Bescheidenheit üben, die Kirche im Dorf lassend somit.Mag sein, dass es eines Tages soweit ist, dass eine viel grössere Theorie da sein wird, dann und nur dann, wird´s gelingen, zu erklären, was ist.Derzeit scheint es mir so zu sein, dass viel zu unüberlegt gehandelt wird, “Träume sind Schäume” und die Theorie der Gravitationswellen scheint dazu zu gehören.Zwar ist es richtig, dass ein Denkmodell da ist, die derzeitige Deutung eingelaufener Signale betreffend, zwar scheint besagtes Denkmodell konsistent zu sein, nicht brüchig, sozusagen, aber es gilt ” eile mit Weile”.Solange man nichts besseres in der Hand hat, gilt es mit dem vorhandenem sich zufrieden zu geben, der Meinung Ausdruck gebend, dass eigentlich genug gesagt worden ist. Denkkonstrukte sind eben Denkkonstrukte.Der Fortschritt von Wissenschaft ist darin zu sehen, dass sich Denkkonstrukte zu entwickeln wissen.

  176. #177 Orci
    23. Februar 2016

    @#170

    So habe ich das damals auch verstanden. Für mich waren das zwei Beispiele von Naturwissenschaft allererster Güte.

  177. #178 Dietmar
    23. Februar 2016

    @Dr. Goltz: Ganz ehrlich: Hast Du noch etwas anderes als Volksmund-Sinnsprüche? Denn Reden ist Silber, Schweigen ist Gold. Da liegt der Hund begraben und der Hase im Pfeffer. Es ist richtig, man sollte das Kind nicht mit dem Bade ausschütten aber der Krug geht nunmal so lange zum Brunnen, bis er bricht. Ich glaube, Du reitest ein totes Pferd.

    Dennoch wirf nicht die Flinte ins Korn! Aber eher geht ein Kamel durchs Nadelöhr als Du damit irgendetwas Überzeugendes sagst.

  178. #179 Spritkopf
    23. Februar 2016

    @Hans Becker

    und gibt mir keinerlei Aufschluss, wie die Jungs bei LIGO und sonstwo die millionenfach größeren Störgrößen aus der Messgröße herausgerechnet haben wollen.

    Bitte sehr:

    Ausführliches zum prinzipiellen Aufbau von LIGO, allerdings noch in der Version von vor 8 Jahren:
    https://arxiv.org/abs/0711.3041

    Über die Maßnahmen, mit denen man bei LIGO die Meßgenauigkeit auf den heutigen Stand gebracht hat:
    https://www.phys.ufl.edu/~tanner/PDFS/Aasi13natphot-squeezing.pdf

  179. #180 Noonscoomo
    Berlin
    23. Februar 2016

    @Goltz
    Irgend wie klngt es, als würdest du die Gravitation als ganzes ablehnen und Gravitationswellen im Speziellen.
    Ok, wir sind uns aber schon einig, dass sich im Wesentlichen die Erde um die Sonne dreht, ja? Irgend was hält also die Erde auf ihrer Kreisbahn. Jetzt stell dir vor, da kommt jetzt ein zweiter Stern angeflogen. Der zieht ja nun auch an der Erde, richtig? Der zweite Stern und unsere Sonne umkreisen sich jetzt sehr eng. D.h. die Erde wird mal von der einen und mal von der anderen Sonne stärker angezogen. Zwischendrin von beiden gleich doll. Wenn wir uns jetzt noch einig darüber sind, dass es “unendlich schnell” nicht gibt, Tadaaaa: eine Welle. Dabei ist erst mal völlig wurst ob Medium oder nicht und wie schnell nun wirklich. Nur aus den beiden Vorraussetzungen.1. es gibt Gravitation und 2. sie ist nicht unendlich schnell ergibt sich: es muss Gravitationswellen geben. Alles andere sind Details. Und alle sind jetzt froh, dass wir was über die Details lernen können. Jetzt, da wir wissen wie es geht können wir neue Geräte bauen die besser sind als die alten.

  180. #181 Captain E.
    23. Februar 2016

    @Dr. Goltz:

    Bitte argumentieren Sie nicht mit Einstein. Nur mal zur Erinnerung: Das war der Physiker, der die spezielle und die allgemeine Relativitätstheorie entwickelt hat. Und in Folge hat er selber die Gravitationswellen postuliert. Er war sich nur von vornherein darüber im Klaren, dass selbst heftigste Verlagerungen von Massen im Universum auf der Erde nur minimale Auswirkungen haben würden. Er hat also nicht die Existenz der Gravitationswellen angezweifelt, sondern nur die Messbarkeit. Und wie wir sehen, hat es tatsächlich etliche Jahrzehnte gedauert, bis die Technik weit genug entwickelt werden konnte.

  181. #182 Florian Freistetter
    23. Februar 2016

    @Hans Becker: “Interessiert Sie das denn überhaupt nicht oder ist die Fragestellung so kompliziert oder trivial, dass ein Physiker die einem Techniker nicht mehr beantworten kann?”

    Die Frage ist: Was genau wollen sie wissen? Wollen sie exakte LIGO-Baupläne sehen? Wie weit sollen die technischen Details gehen? Jahre- bzw. Jahrzehntelange Konstruktionsarbeit kann man nicht eben mal in einem Kommentar erläutern. Es gibt hunderte Störquellen und viele, viele unterschiedliche Strategien um damit klar zu kommen. Die alle aufzuschreiben würde ein (vermutlich eher langweiliges) Buch nötig machen. Ein wichtiger Punkt ist sicherlicher die Trennung der Observatorien durch 3000 km. Was nicht bei beiden zu sehen ist, ist eine Störung. Aber die Details die sie interessieren kann ihnen wohl nur ein LIGO-Ingenieur verraten…

  182. #183 Alderamin
    23. Februar 2016

    @Hans Becker, Spritkopf

    Noch ausführlicher ist dieser Artikel, der geht auf alle Störungen ein. Auch, dass man die Signale durch IIR-softwarefilter aus dem Rauschen herauszieht (klar, was sonst?). Das Gerät ist halt für einen schmalen Frequenzbereich von einigen 10 bis 100 Hertz ausgelegt, den man für verschmelzende kompakte Objekte oder Kernkollaps-Supernovae erwartet.

  183. #184 Krypto
    23. Februar 2016

    @Dietmar:
    Der war gut!
    Dabei stört mich das hochgestochene Geschwafel noch nicht einmal, es unterbindet recht schnell das Weiterlesen 😀
    An den Anführungszeichen bin ich jedoch hängen geblieben:

    “ART”

    Ah…von da weht der Wind… 😉

    @Hans Becker:
    Sollen wir für Dich googlen? 😉

  184. #185 Dr. Goltz
    23. Februar 2016

    Vielen Dank für die Anregungen, welche gegeben worden sind.Ich bin grad dabei eine Brücke zu bauen zwischen Atomphysik und der Gravitationslehre, dies am Rande.Vielen Dank für Aufmerksamkeit.

  185. #186 PDP10
    23. Februar 2016

    @Dr. Goltz:

    “Ich bin grad dabei eine Brücke zu bauen zwischen Atomphysik und der Gravitationslehre, dies am Rande.”

    Oh! Und wir haben als erste davon erfahren!

    Das bedeutet doch wohl hoffentlich, dass wir alle zu den Festivitäten rund um die Nobelpreisverleihung eingeladen werden, oder?

  186. #187 Higgs-Teilchen
    Im Standardmodell oben rechts
    23. Februar 2016

    @Dr. Goltz
    “Nimmt man an, dass es Gravitationswellen gibt, macht man vermutlich einen Fehler, darauf hinweisend, dass selbst Einstein in dieser Richtung schon Zweifel gehegt hat”

    Na und? Robert Oppenheimer hatte am Anfang auch daran gezweifel, ob man den Atomkern spalten kann. Hatte das sogar vorgerechnet. Dass er damit falsch lag hat er danach aber eingesehen – und in der Praxis auch bewiesen, dass es sehr wohl funktioniert.

    Lg H.

  187. #188 noonscoomo
    Berlin
    23. Februar 2016

    @Goltz
    Und Max Planck hat sehr lange versucht, das Wirkungsquantum wieder los zu werden, weil er die klassische Physik retten wollte.

  188. #189 Captain E.
    24. Februar 2016

    Tja, so langsam sollten es wirklich einsehen: Das Universum ist nicht nur eine “Scheißgegend” (laut Science Busters-Bühnenshow), sondern auch total und völlig verrückt. Jeder Wissenschaftler, der bekloppt genug ist, eine verrückte Idee zur Beschreibung des Universums zu formulieren, muss damit rechnen, dass er nicht nur ein klein wenig besser liegt als seine Vorgänger, sondern dass seine Nachfolger noch auf sehr viel verücktere Ideen kommen werden – die dann noch besser das Universum beschreiben. Und wenn es ganz blöd läuft, werden die eigenen Ideen von den lieben Kollegen auch noch gegen einen verwendet. Einstein ist bekanntlich genau so etwas passiert…

    Wie gesagt: Das Universum ist total und völlig verrückt. Wenn sich das jemand wirklich ausgedacht hat, gehört derjenige mindestens bis zum Ende der Ewigkeit in eine geschlossene Anstalt.

  189. #190 PDP10
    24. Februar 2016

    Jetzt war ich doch neugierig und wollte mir mal dem Dr. Goltzen seine in #125 verlinkte Heimatseite ansehen …

    Allerdings gibt es dort nur eine komplett leere Seite zu sehen. Jedenfalls mit meinen Browser-Einstellungen.
    Mein Browser fragt mich nämlich umgehend “Das Ausführen von Flash erlauben?”

    Also wenn man ohne Flash so gar nix sieht?

    Nö.

  190. #191 Spritkopf
    24. Februar 2016

    @PDP10
    Die Flash-Seite ist nur der Einstieg, danach gehts mit HTML weiter. Aber glaub mir, das willst du nicht sehen.

  191. #192 PDP10
    24. Februar 2016

    @Spritkopf:

    “Aber glaub mir, das willst du nicht sehen.”

    Das habe ich mir nach dem Einstieg schon gedacht :-).

  192. #193 Dr. Goltz
    25. Februar 2016

    http://www.evolution-of-nature-in-german.com/ , meine kostenpflichtige Seite, wird derzeit in aller Welt aufgerufen, in den USA, in China usw., der Länderstatistik folgend.

  193. #194 Dr. Goltz
    25. Februar 2016

    Im März dieses Jahres werde ich die letzten Einträge in den Blog meiner Homepage einsetzen, als letzten Eintrag darauf hinweisend, wie man mich bei Facebook erreichen kann, weiter oben schon angegeben.Der Sinn meiner Facebookseite besteht darin, dass kräftig gesiebt wird, die Vielzahl der Homepagedarstellungen betreffend.

  194. #195 Dr. Goltz
    25. Februar 2016

    Natürlich ist es auch erforderlich, daran zu denken, dass meine Homepageseite einst aus dem Netz verschwindet, sie ist kostenpflichtig. Die Facebookseite von mir wird lange, lange Zeit im Netz bleiben, vielleicht über 20, 30 Jahre und so fort.Mag sein, dass sie noch in 50 Jahren im Netz zu finden sein wird.Entsprechend müssen die Aussagen meiner Facebookseite sein.

  195. #196 Dietmar
    25. Februar 2016

    Jetzt hast Du toll Werbung gemacht für Dein Hypothesen. Eindruck würdest Du aber nur machen, wenn Du daraus seröse Arbeiten machen würdest, wissenschaftlich veröffentlicht. Warum Du das nicht machst, dürfte klar sein, egal, wie Deine Ausreden aussehen.

    Mein Kanal wurde schon in USA oder auch Japan aufgerufen.Ca. 30 mal in 6 Jahren. ICH BIN WELTSTAR!!!1!11!!!

  196. #197 Spritkopf
    25. Februar 2016

    @Dr. Goltz

    Die Facebookseite von mir wird lange, lange Zeit im Netz bleiben, vielleicht über 20, 30 Jahre und so fort.Mag sein, dass sie noch in 50 Jahren im Netz zu finden sein wird.Entsprechend müssen die Aussagen meiner Facebookseite sein.

    Ok, zu den religiösen, ähm, “Vorstellungen”, zu den paranoiden Anwandlungen und der Crackpottery, die auf der Seite zu finden sind, kommt noch ein übersteigertes Geltungsbedürfnis.

    Bei allem Respekt: Suchen Sie sich einen Therapeuten. Das ist nicht herabwürdigend gemeint, sondern ernst!

  197. #198 Captain E.
    25. Februar 2016

    Ob Facebook überhaupt noch so lange bestehen wird? Das Unternehmen stellt ein Produkt zur Verfügung, was seit einigen Jahren modern ist, aber über Moden geht die Zeit gnadenlos hinweg.

  198. #199 Eisentor
    25. Februar 2016

    Die Facebookseite von mir wird lange, lange Zeit im Netz bleiben, vielleicht über 20, 30 Jahre und so fort.Mag sein, dass sie noch in 50 Jahren im Netz zu finden sein wird.

    das haben wir bei GeoCities oder StudiVZ auch gedacht 😉

  199. #200 Dr. Goltz
    25. Februar 2016

    Jetzt geht es beinahe in´s lustige.Wann immer ich hierzulande behaupte, dass ich etwas weiss, erfahre ich wie folgt Anfeindungen: es gehört sich nicht etwas zu wissen usw..Ich habe beispielsweise herausgefunden, dass die fernen Sterne, die, die ausserhalb des Planetensystems sind, näher zur Erde sind, wie die dogmatische Lehrbuchliteratur es zu behaupten weiss.

  200. #201 Dr. Goltz
    25. Februar 2016

    In meinem gesamten Bekanntenkreis ist es Sitte geworden, mir beinahe andächtig zuzuhören.Ich trenne zwischen Bekannten, Freunden usw.,.da man sich, jeweils nach Denkrichtung, mal so, mal so äussert, wenn überhaupt eben,gehe ich dazu über, alles bei Facebook unterzubringen, was Aufmerksamkeit erheischen könnte.

    Es gibt ja neuerdings das Fachgebiet Wissenschaftskabarett, in dieser Richtung scheine ich nicht gänzlich unbegabt zu sein.

    Nur Mut in allem, es geht weiter.

  201. #202 Eisentor
    25. Februar 2016

    Ich habe beispielsweise herausgefunden, dass die fernen Sterne, die, die ausserhalb des Planetensystems sind, näher zur Erde sind, wie die dogmatische Lehrbuchliteratur es zu behaupten weiss.

    Wie haben sie das herausgefunden? Nur nachdenken ist leider nicht ausreichend.

  202. #203 Captain E.
    25. Februar 2016

    Nein, das haben Sie nun völlig falsch verstanden. Selbstverständlich dürfen Sie etwas wissen. Sie dürfen sogar etwas wissen, was sich vom Wissen anderer Leute unterscheidet. Nur wundern Sie nicht bitte nicht, wenn diese anderen Leute von Ihnen Belege erwarten. Beweise erwartet niemand von Ihnen, weil es so etwas in den Naturwissenschaften mit Ausnahme der Mathematik prinzipiell nicht gibt. Geben Sie den anderen etwas zum Herumspielen, etwas, dass man möglicherweise widerlegen kann. Wenn es trotz großer Anstrengungen nicht widerlegt werden kann, ist es brauchbar. Wenn es überhaupt keine Möglichkeit gibt, Ihre Idee zu falsifizieren, dann taugt sie schlichtweg nichts.

  203. #204 Dr. Goltz
    25. Februar 2016

    Mal als Clown arbeitend, mal nicht,kommt es derzeit nicht in Frage, wissenschaftlich etwas zu veröffentlichen.Ich bin ja gehalten Geld zu verdienen, was ich als Clown tue. Ich wüsste keinen Verlag, der bereit wäre, jemand zu stützen, der clowneresk ist, einerseits, dann wieder bierernst usw..

  204. #205 Captain E.
    25. Februar 2016

    @Dr. Goltz:

    In meinem gesamten Bekanntenkreis ist es Sitte geworden, mir beinahe andächtig zuzuhören.Ich trenne zwischen Bekannten, Freunden usw.,.da man sich, jeweils nach Denkrichtung, mal so, mal so äussert, wenn überhaupt eben,gehe ich dazu über, alles bei Facebook unterzubringen, was Aufmerksamkeit erheischen könnte.

    Wenn die Leute einem nur noch andächtig zuhören, ist das ein ernster Warnhinweis. Da sollten Sie noch einmal in sich gehen.

    Es gibt ja neuerdings das Fachgebiet Wissenschaftskabarett, in dieser Richtung scheine ich nicht gänzlich unbegabt zu sein.

    Nur Mut in allem, es geht weiter.

    Leider ist die Begabung hier noch nicht offenbart worden. Was Sie hier schreiben, ist nicht besonders wissenschaftlich und dabei noch nicht einmal amüsant.

  205. #206 Spritkopf
    25. Februar 2016

    @Eisentor

    Wie haben sie das herausgefunden?

    Das steht auf seiner Seite. *hust*

  206. #207 Dr. Goltz
    25. Februar 2016

    Mit Tausenden von Menschen in den letzten 10 Jahren gesprochen habend, gab es fast nie, eigentlich nie, die Möglichkeit meinen Theoriebeweis zur relativen Nähe ferner Sterne im Gespräch zu halten.Während ich besagten Beweis, es ist kein Beweis im allerstrengstem Sinne, mehr ein Pseudobeweis, quasi- entwickelte,war ich in allen Einzelheiten drin, der besagte lerndogmatisch zu nennende Beweis wurde vor knapp 7 Jahren von mir im Internet dargestellt, in so vielen Einzelheiten, wie erforderlich war, siehe meine Homepage http://www.evolution-of-nature-in-geman.com/

  207. #208 Captain E.
    25. Februar 2016

    @Dr. Goltz:

    Mal als Clown arbeitend, mal nicht,kommt es derzeit nicht in Frage, wissenschaftlich etwas zu veröffentlichen.Ich bin ja gehalten Geld zu verdienen, was ich als Clown tue. Ich wüsste keinen Verlag, der bereit wäre, jemand zu stützen, der clowneresk ist, einerseits, dann wieder bierernst usw..

    Da kann man es fast schon verstehen, dass manche Menschen Angst vor Clowns haben. Aber ist es nicht eigentlich für einen Clown ein Sakrileg, die Menschen nicht zu amüsieren? Wie schon gesagt: Ein besonders amüsanter Clown sind Sie nicht gerade.

  208. #209 Captain E.
    25. Februar 2016

    @Dr. Goltz:

    Mit Tausenden von Menschen in den letzten 10 Jahren gesprochen habend, gab es fast nie, eigentlich nie, die Möglichkeit meinen Theoriebeweis zur relativen Nähe ferner Sterne im Gespräch zu halten.Während ich besagten Beweis, es ist kein Beweis im allerstrengstem Sinne, mehr ein Pseudobeweis, quasi- entwickelte,war ich in allen Einzelheiten drin, der besagte lerndogmatisch zu nennende Beweis wurde vor knapp 7 Jahren von mir im Internet dargestellt, in so vielen Einzelheiten, wie erforderlich war, siehe meine Homepage

    Ich habe es oben schon gesagt: Einen Beweis erwartet niemand von Ihnen, nur ein paar handfeste Belege, an denen man sich abarbeiten kann. Wenn die sich als belastbar herausstellen, also niemand sie widerlegen kann, dann haben Sie etwas entdeckt. Ihre Pseudobeweise können sie also stecken lassen, denn so etwas ist grundsätzlich nicht falsifizierbar – und somit wertlos.

  209. #210 Dr. Goltz
    25. Februar 2016

    Im Rahmen meiner mit “sehr gut” abgeschlossenen Doktorabeit, habe ich das Problemlösevermögen von Ingenieurstudenten zu steigern gewusst.Nach wie vor befinde ich mich auf dieser Linie, jetzt das Problemlösevermögen von Diplom-Ingenieuren etwa, steigern wollend, anmerkend, dass man astronomische Entfernungen, die, von denen hier die Rede ist, ausserhalb des Schwerefeldes der Erde zu tätigen hat, etwa auf einem Kometen.Herausgefordert sind also Techniker.

  210. #211 Dr. Goltz
    25. Februar 2016

    Ich neige etwas dazu, zu bescheiden zu sein.Mein Beweis
    zur Nähe ferner Sterne ist so genau, wie es nur irgendwie ging, viele Jahre daran gearbeitet habend, Jahrzehnt auf Jahrzehnt beinahe.Um meinen Beweis zu verstehen, er ist ja im Netz zu finden, bedarf es einiger Stunden Denkarbeit.

    Bitte verzeihen Sie mir, dass ich jetzt mich in anderer Richtung zu sputen habe.Es dreht sich darum, dass mein Beweis weltbekannt wird, nur so ist denkbar, dass irgendwann Geld mobilisiert wird,um die jetzigen Messdaten zur Entfernung ferner Sonnen zu überprüfen, mittels Raumschifftechnik also.

    Verzeihen Sie mir bitte, ich wollte nicht stören.

  211. #212 Eisentor
    25. Februar 2016

    Sie glauben also das außerhalb des “Schwerefeldes” der Erde die Entfernungen anders gemessen werden? Was ist mit den Satelliten die wir dort schon haben? Die Lagrange-Punkte sollten doch so weit weg sein das eine merkliche Änderung bemerkbar währe?

  212. #213 Bullet
    25. Februar 2016

    Ich wollte mich ja heraushalten, aber:

    Ich habe beispielsweise herausgefunden, dass die fernen Sterne, die, die ausserhalb des Planetensystems sind, näher zur Erde sind, wie die dogmatische Lehrbuchliteratur es zu behaupten weiss.

    Das wäre mir neu. Schließlich war die , äh, “dogmatische Lehrbuchmeinung” lange Zeit diese. (Übrigens: wer dem Link folgt und dort einen Erdradius von “859 Meilen” liest: eine Deutsche Meile sind lt. wiki 7,42 km – das paßt also einigermaßen.)
    Die, äh, “dogmatische Lehrbuchmeinung” mußte eben genau wegen dieser blöden neuen Spielzeuge wie besseren Teleskopen, Astrofotografie, Spektroskopie et al. fallengelassen werden.
    Aber was red ich… wer

    1-a) Die jetzige Lehrmeinung zur Entfernung sehr ferner Sterne, den sogenannten Fixsternen, ist derzeit noch etabliert.

    schreibt, der muß sich fragen lassen, was denn dann bitte nicht so ferne Sterne sind, und warum die nicht fix sein sollen (zumindest nicht so fix wie die fernen…) – moah, was für ein Gestammel.
    Das geilste aber ist in einer Grafik versteckt: “CO2-Partikel-Gravitation lenkt vermehrt solare Photonen zu uns”.
    Auf DIE Idee muß man erstma’ kommen. Und nein, das ist kein gutes Zeichen…
    Dr. Goltz:

    MAN IST LEHRBUCHMÄSSIG ÜBERZEUGT, DASS DIE NÄCHSTEN FIXSTERNE SO UND SO WEIT ENTFERNT SIND, HÄLT ES FÜR WAHR UND ÜBERSIEHT, DASS ES SICH UM “TATSACHENUNTERSCHLAGUNGEN” HANDELT.

    Klar. Alle haben es übersehen, nur du nicht. Was meinst du denn, woher diese “Lehrbuch”-Zahlen kommen? Und viel schlimmer: wie kommst du auf die Idee, daß deine (ernsthaft extrem kruden) Behauptungen irgendwas mit der Wirklichkeit zu tun haben? Wie hast du deine Ideen überprüft?

  213. #214 Captain E.
    25. Februar 2016

    @Dr. Goltz:

    In welchem Studium schreibt man denn Doktorarbeiten über das Steigern des Problemlösevermögen von Ingenieurstudenten? Spontan würde ich auf Psychologie tippen. Nur wie qualifiziert Sie das dazu, Ihr eigenen physikalischen Hypothesen aufzustellen? Und noch einmal zur Erinnerung: Sie haben keinen Beweis! Sie können nämlich deshalb keinen haben, weil es so etwas wie Beweise in der Physik und der Astronomie gar nicht gibt. Liefern Sie uns Belege, die sich prinzipiell widerlegen ließen, und wir kommen ins Geschäft. Nur wenn Ihre Hypothesen und die beigebrachten Belege sich trotz großer Anstrengung nicht widerlegen lassen, haben Sie vielleicht etwas von Wert entdeckt. Etwas, dass sich nicht überprüfen lässt, ist schlichtweg wertlos.

  214. #215 Bullet
    25. Februar 2016

    Ach, er geht schon wieder, seine vor Weisheit triefende Botschaft hinterlassen habend, auf daß sie die Saat in alle Welt trage.
    Na dann Prost.

  215. #216 Captain E.
    25. Februar 2016

    Tja, früher waren Clowns auch mal lustig und haben mit völliger Berechtigung Geld für ihre Darbietung bekommen. Heute gibt es anscheinend welche, die stinklangweilig sind und sich für selbst gezimmerte pseudowissenschaftliche Weltbilder bezahlen lasen möchten.

  216. #217 Dr. Goltz
    25. Februar 2016

    Was ich nicht verstehe ist dies: ungemein clowneresk gekleidet seiend, erhalte ich fast kein Geld für besagte Arbeit.

    Es ist zu unterscheiden zwischen der Methode des Geldverdienstes und der Arbeit als solcher.

    Im Rahmen dieser Ausführungen war ich bestrebt “zur Sache” zu kommen, arbeite ich als Clown gibt es viele Lacherfolge.

    Das eine ist das eine, das andere, das andere.

  217. #218 Eisentor
    25. Februar 2016

    @Dr. Goltz wollen sie ein bisschen auf die Fragen eingehen die die Leute zu ihren Ideen haben?

  218. #219 Dr. Goltz
    25. Februar 2016

    Tatsächlich bin ich Denkpsychologe.Zuerst Elektrotechnik studiert habend, wechselte ich dann die Fachrichtung, also Dipl.-Ing.Dr.phil. seiend. Abwechslungshalber, nur so gelingt es mir vergnügt zu bleiben, arbeite ich quasi als Clown, nun wirklich abschliessen müssend.Ein jeder hat Gönner, wie Gönnerinnen, ich nähmlich auch.Jegliches Geschäftsgeheimnis sollte nun wirklich nicht ausgeplaudert werden.

    Leben Sie wohl.

  219. #220 Florian Freistetter
    25. Februar 2016

    @Dr. Goltz: “Im Rahmen dieser Ausführungen war ich bestrebt “zur Sache” zu kommen, arbeite ich als Clown gibt es viele Lacherfolge.”

    Zur Sache ist ein gutes Stichwort. All das hat nix mit Gravitationswellen zu tun. Für sowas gibts den Offtopic-Bereich.

  220. #221 jolietjake
    25. Februar 2016

    Natürlich war das Offtopic. Aber zumindest ich bin auf’s köstlichste amüsiert worden. Bester Troll, pardon, Clown seit Langem.
    In bester Stimmung seiend bedanke ich mich herzlichst!

  221. #222 Krypto
    25. Februar 2016

    @Florian:
    Ich sehe da schon einen Bezug und stelle die Vermutung auf, dass GW in passend geformten Schädeln übelste Resonanz-Schäden hervorrufen können. 😉
    Mich ebenfalss köstlich amüsiert habend, hege ich eine weitere Vermutung:
    Mag es angehen, dass sich Dr. Goltz seinerzeit als perfekte deutsche Synchronstimme für Meister Yoda erfolgreich beworben habend gehabt hat?

  222. #223 jolietjake
    25. Februar 2016

    Synchronstimme?
    “Dr.” Goltz ist Meister Yoda. Oder Yoghurt. Äh, war er gewesen seiend.
    Für mich beinahe auf einer Stufe mit Oberguru Sheng Fui!

  223. #224 Krypto
    25. Februar 2016

    “Direkt nachgewiesen Gravitationswellen wir haben.”

    *duck und halb-OT weg*

  224. #225 PDP10
    25. Februar 2016

    Das war seltsam … Dr. Phil. Seltsam sozusagen …

  225. #226 Dr. Goltz
    26. Februar 2016

    Die Kritik geschah völlig zu recht.Ich bin nicht am Thema der Gravitationswellen geblieben, was mir sehr leid tut.Vielleicht finde ich jemand, der mich über mein Thema, mein jetziges Thema, dass der Thematik der Quantengravitation, endlich nachdenken lässt, mutmassend, dass anisotrope Verhältnisse thermischer Art einen hohen Stellungswert hinsichtlich der quantengravitativen Bewertung gravitationswellenmechanischer Dingheiten haben dürften.Dies nur am Rande.

  226. #227 Dr. Goltz
    26. Februar 2016

    Es fiel mir einiges ein zu besagter Thematik, dies aber jetzt nicht ventilieren dürfend, da ich nur über eine Konzeptstudie sozusagen verfüge.Jetzt wieder zu meiner Arbeit als Clown aufbrechen müssend, von nichts kommt ja nichts, alles kostet was, umsonst ist nur der Tod, gilt es, strenges stillschweigen zu bewahren, nicht jeder interessiert sich für das Thema der Thermik im Sachgebiet des Versuches, Gravitationswellen nachzuweisen, noch darauf hinweisend, dass es sich um die Unterscheidung zwischen echtem und unechtem nichts handelt u.a., nicht nur natürlich, wichtig ist auch die Thematik des Antigravitons, des von mir entdeckten Feldes der Antigravitonen, welche im Gegensatz zu sehen sind zum Feld der von mir erkannten Gravitonen.

  227. #228 Dr. Goltz
    26. Februar 2016

    Im vorletzten Archivanteil des Blogs von mir,per “Urknall der Weisheit” aufrufbar, etwa, habe ich Grundzüge einer Konzeptstudie zur quantenquantitativen Begründung meiner Grundthesen darzustellen gewusst.”Es ist noch kein Meister vom Himmel gefallen”, allerdings und die Arbeit geht weiter, gelegentlich als Clown, aber beileibe nicht immer.

  228. #229 Spritkopf
    26. Februar 2016

    @Goltz
    Ihr gespreiztes und möchtegern-intellektuelles Geschwafel über Ihre Privatphysik nervt.

  229. #230 Captain E.
    26. Februar 2016

    Entdeckung der Gravitonen und sogar der Antigravitonen? Dazu gibt es doch sicherlich ein peer-reviewtes Paper, oder? Denn falls nicht ==> keine Entdeckung.

  230. #231 Kyllyeti
    26. Februar 2016

    ”Es ist noch kein Meister vom Himmel gefallen”

    Da seht ihr’s doch – das schaffen die doch wohl nur mit Antigravitonen, oder?

  231. #232 noch'n Flo
    Schoggiland
    26. Februar 2016
  232. #233 Adent
    26. Februar 2016

    Also meines Erachtens verarscht Euch der Dr. Goltz und feixt sich einen. Die Aneinanderreihung von nichtssagenden Platitüden a la “Das Kunstwerk osszilliert im Spannungsfeld zwischen diskursiver Eloquenz und ironischem Zitat” (auf jeder Vernissage anwendbar) und der ständige Verweis darauf, er müsse wieder als Clown arbeiten (gefolgt von 3-5 Nullaussagen wie Von Nichts kommt Nichts etc.) ist doch recht eindeutig findet ihr nicht?
    Also Herr Goltz, amüsieren sie sich gut? Wenn ja, fein gemacht, wenn nein dann gehen Sie bitte zu einem Arzt ihres Vertrauens.

  233. #234 Spritkopf
    26. Februar 2016

    @Adent

    Also meines Erachtens verarscht Euch der Dr. Goltz und feixt sich einen.

    Wenn ich nach seiner Webseite gehe, dann nicht.

  234. #235 Adent
    26. Februar 2016

    @Spritkopf
    Echt nicht, oh Gott…

  235. #236 Vortex
    26. Februar 2016
  236. #237 Higgs-Teilchen
    Im Standardmodell oben rechts
    26. Februar 2016

    @ noch’n Flo

    Was ist mit deinem Avatar passiert? 🙂

  237. #238 Krypto
    26. Februar 2016

    @Dr. Goltz:
    Meiner Meinung nach solltest Du dringend mit einem Arzt Deines Vertrauens reden über die Dinge, die Du im Internet veröffentlichst.
    Ich gehe nicht davon aus, dass Du Dir nur einen Spaß daraus machst.

  238. #239 noch'n Flo
    Schoggiland
    26. Februar 2016

    @ Higgs-Teilchen:

    Was ist mit deinem Avatar passiert?

    Die Blogplattform hatte in den letzten Tagen immer wieder Schluckauf, so dass ein paar Kommentare ärgerlicherweise auf Nimmerwiedersehen entschwanden. Seltsamerweise hilft dagegen manchmal, Nich oder eMail-Addy leicht abzuändern. Hab nur vergessen, das Auto-Fill-In wieder zurückzusetzen.

  239. #240 PDP10
    26. Februar 2016

    @Adent, Vortex etc.:

    Hmmja … ich neige ja eigentlich inzwischen auch zu der Ansicht, dass wir es hier mit einem Scherz oder Kunst zu tun haben.

    Aber ihr wisst ja:

    Immer wenn hier einer von den Vögeln auf der Durchreise vorbeigekommen ist und man denkt verrückter gehts nicht …

    … kommt noch einer.

  240. #241 Dampier
    26. Februar 2016

    Die Infografiken sind eindeutig Kunst. Ich muss sagen, dass ich Dr. Goltz’ Seiten mit einer gewissen Faszination gelesen habe.

  241. #242 Dr. Goltz
    27. Februar 2016

    Tatsächlich handelt es sich um meinen Versuch, etwas zur Lachkultur beizutragen, ich habe seit Jahrzehnten nicht mehr so viel gelacht, manche Einträge gelesen habend.Dessen ungeachtet bleibt der sachliche Gehalt meiner Eintragungen.Nur mit Humor lässt sich das Leben meistern, finde ich.

  242. #243 noch'n Flo
    Schoggiland
    27. Februar 2016

    @ Dr. Goltz:

    Tatsächlich handelt es sich um meinen Versuch, etwas zur Lachkultur beizutragen

    Das ist löblich. Und mit Kommentaren wie beispielsweise

    Dessen ungeachtet bleibt der sachliche Gehalt meiner Eintragungen.

    gelingt Ihnen das auch hervorragend. Respekt!

    Oh, und natürlich: ROFL!!!

  243. #244 Florian Freistetter
    27. Februar 2016

    Ich würde mich immer noch freuen, wenn die Diskussion über die Homepage eines Trolls im Offtopic-Bereich stattfinden würde.

  244. #245 Spritkopf
    27. Februar 2016

    @Goltz #242

    Dessen ungeachtet bleibt der sachliche Gehalt meiner Eintragungen gleich Null.

    Korrektur eingefügt habend.

  245. […] möchte ich auch die Lektüre von Florian Freistetters Artikeln zum Thema Gravitationswellen hier und hier ans Herz legen. Sehr […]